Uworld incorrect week first few exams till nbm2 16

¡Supera tus tareas y exámenes ahora con Quizwiz!

Histological features in ischemic brain disease/stroke according to time frame.

12-24HRS: Eisinophilic cytoplasm+pyknotic nuclei (red nucleus) 24-72hrs: Necrosis + neutrophils 3-5days: Macrophages (microglia) 1-2weeks: Reactive gliosis (astrocytes) + vascualar proliferation >2weeks: Glial scar

A 62-year-old woman comes to the office for medical evaluation. The patient states her 26-year-old son has developed renal failure from chronic glomerulonephritis and is undergoing dialysis therapy. She has the same blood group as her son and she wants to donate one of her kidneys to him. The patient has no chronic medical conditions and takes no medications. Vital signs are within normal limits, and physical examination shows no abnormalities. The patient understands that she may not be a compatible donor based on human leukocyte antigen (HLA) testing. In addition, which of the following age-related renal changes should be taken into consideration when assessing donor suitability?

Decreased number of functional glomeruli Normal aging results in multiple physiologic and structural alterations that lead to a slow decline in renal function, even in the absence of renal disease. These alterations typically begin around age 30, with a more marked decline occurs in those >50 years, resulting in the predisposition of elderly patients to develop acute kidney injury or chronic kidney disease. These alterations include: Reduced renal mass and functional glomeruli: There is a 50% reduction in functional glomeruli by age 75 associated with a reduction in renal mass due to atrophy and fibrotic replacement. This results in reduced glomerular filtration rate and creatinine clearance, as well as a reduced ability to concentrate urine, which may predispose patients to hypovolemia during periods of stress (Choice C). Reduced renal blood flow (RBF): Loss of renal microvasculature results in a reduction in renal blood flow with age, increasing the susceptibility to ischemic injury (Choice D). Furthermore, RBF becomes more dependent upon prostaglandins to maintain adequate blood flow, leading to increased susceptibility to renal injury with nonsteroidal anti-inflammatory drugs (due to reduced prostaglandin formation). Reduced hormonal responsiveness: There is reduced secretion of renin (blunts the renin-angiotensin-aldosterone system response) and reduced hydroxylation of vitamin D in response to parathyroid hormone (Choice E). However, unlike chronic renal disease, the production of erythropoietin in response to anemia or hypoxemia is unchanged. Older kidneys also have a higher proportion of cells that undergo apoptosis after a given insult (eg, ischemia) and are less capable of regeneration. (Choice B) The ability to excrete solutes is relatively preserved in the aging kidney.

TDT+ IN WHICH LEUKEMIA/LYMPHOMA? MPO+?

TDT+ IN ACUTE LYMPHOBLASTIC LEUKEMIA MPO+ AML

A 64-year-old man is evaluated for persistent fever and weakness. He has a history of aortic valve replacement for aortic stenosis. Physical examination reveals a new cardiac murmur with scattered petechiae and splinter hemorrhages seen on his extremities. Echocardiogram shows a vegetation involving one of the aortic valve leaflets, and blood cultures grow enterococci. As part of the patient's treatment, 240 mg of intravenous gentamicin is started. The pharmacy calculates that, in this patient, gentamicin has a volume of distribution of 30 L, a half-life of 4 hours, and demonstrates first-order and one-compartment kinetics. Which of the following is the most likely serum drug concentration just before the next dose 8 hours later?

2mg/dl 240/30 now do the half life shit 8 after 4 hrs will be 4 and 8hrs 2

A 34-year-old, previously healthy woman comes to the office due to irregular menstruation and a 4.5 kg (10-lb) weight gain over the past 6 months. The patient also reports muscle weakness causing difficulty climbing stairs or lifting her child. Blood pressure is 152/86 mm Hg and pulse is 72/min. BMI is 31 kg/m2. Physical examination shows excessive fat accumulation in the abdomen and supraclavicular area. Sun-exposed areas and skin creases are hyperpigmented and there are wide striae over the abdomen. Coarse hair is present on the face, chest, and abdomen. Which of the following sets of laboratory findings are most likely to be seen in this patient? 24hr urine cortisol? acth? and DHEAS?

ACTH INCREASE CORTISOL INCREASE AND DHEAS TOO CUSHING DISEASE Cushing syndrome can be categorized as ACTH-dependent or ACTH-independent. Excess ACTH secretion in ACTH-dependent Cushing syndrome leads to hyperpigmentation. In addition to increased cortisol production, ACTH stimulates production of adrenal androgens (eg, testosterone, dehydroepiandrosterone sulfate) leading to hirsutism and menstrual irregularities.

A 23-year-old asymptomatic male participates in clinical research and is found to be homozygous for the apolipoprotein E-4 allele. In the future, this patient is most likely to suffer from which of the following?

Alzheimer disease has a strong genetic predisposition. About 30% of patients have a family history of the disease. According to age when symptoms begin, Alzheimer disease can be classified as early or late. The following three mutation sites have been associated with early-onset familial Alzheimer disease (onset <60 years old): Amyloid precursor protein (APP) gene on chromosome 21 Presenilin 1 gene on chromosome 14 Presenilin 2 gene on chromosome 1 Both APP and presenilin gene mutations are thought to promote the production of A β-amyloid. Late-onset familial Alzheimer disease is associated with the ε4 allele of Apolipoprotein E. The exact mechanism of its influence is not known. It is thought that the ApoE4 protein may be involved in the formation of senile plaques.

A 62-year-old man is brought to the emergency department from a homeless shelter due to alcohol intoxication. The patient has no other symptoms or history of significant medical conditions. Examination of the scalp reveals the findings shown in the exhibit. Which of the following organisms is most likely responsible for this patient's condition? A) malesazzi B) scabies C)Pediculus Humanus Capitis d) Trichopythan

C head lice

A 36-year-old man comes to the office due to skin lesions on his palms. The patient has yellowish skin nodules over the palmar creases that have been increasing in size and number over the past several years. He also has small clusters of yellow papules on his elbows, knees, and buttocks. His father died of a myocardial infarction at age 56. Biopsy of his lesions shows accumulation of lipid-laden macrophages. Immunoblot analysis suggests a lack of ApoE3 and ApoE4 in his circulating lipoproteins. Which of the following is most likely impaired in this patient? Whats the dx and whats the shit thats going to be high?

Familial dysbetalipoproteinemia (type III hyperlipoproteinemia) is characterized by xanthomas and premature coronary and peripheral vascular disease. It is an autosomal recessive disorder that is clinically more severe in patients with other conditions affecting lipoprotein metabolism (eg, diabetes, hypothyroidism). The primary defects in familial dysbetalipoproteinemia are in ApoE3 and ApoE4, apolipoproteins found on the triglyceride-rich lipoproteins (chylomicrons and VLDLs) that are responsible for binding hepatic apolipoprotein receptors. Without ApoE3 and ApoE4, the liver cannot efficiently remove chylomicrons and VLDL remnants from the circulation, causing their accumulation in the blood and resultant elevations in cholesterol and triglyceride levels.

A 20-year-old woman comes to the urgent care center due to 3 days of vaginal pain with intercourse and dysuria. She also had low-grade fevers and a mild headache. The patient is otherwise healthy and takes oral contraceptives only. She has no history of urinary tract infections or sexually transmitted infections. She is sexually active and recently began a monogamous relationship with a new partner. Her boyfriend uses condoms inconsistently. On examination, she is afebrile. She is uncomfortable, but in no acute distress. Her abdomen is soft and nontender. Genital examination reveals multiple, painful, shallow ulcers with an erythematous base on the left labia. There is no vaginal discharge. Internal examination reveals no cervical motion tenderness. She has bilateral tender inguinal lymphadenopathy. Which of the following is the best diagnostic test for this patient's condition? DX?

Genital herpes tzanks smear or DNA pcr Primary genital herpes simplex virus (HSV) infection is the most likely diagnosis in this patient who has multiple painful genital ulcers (with a characteristic erythematous base), dysuria (likely due to irritation from the ulcers), tender lymphadenopathy (common with primary genital HSV infection), and systemic symptoms (fever, headache) in the setting of having a new sexual partner. The characteristic vesicles are often absent. HSV infection can be diagnosed using polymerase chain reaction (PCR), direct fluorescence antibody, viral culture (low sensitivity), or Tzanck smear (showing multinucleated giant cells).

A scientist is interested in the mechanisms by which leukocytes traffic to sites of inflammation and infection. She finds that endothelial cells increase the expression of certain cell surface molecules in response to cytokines to allow for leukocyte trafficking. She subsequently creates a knockout mouse that has a deletion in the platelet endothelial cell adhesion molecule 1 (PECAM-1) gene. The protein product of this gene is mainly localized to specific areas on the endothelial cells. Absent expression of this gene will most likely affect which of the following neutrophil functions? 1) margination 2) rolling 3)adhesion 4)diapedesis/transmigration 5)migration

Inflammation is an important process in the defense against various pathogens. It is characterized by the passage of leukocytes into the inflamed tissue. The following steps are involved in inflammatory leukocyte accumulation: Margination: Increased vascular leakage in the microvasculature leads to hemoconcentration and decreased wall shear stress, improving the contact of neutrophils with the endothelial lining Rolling: Neutrophils roll on the endothelium via the loose binding of sialylated carbohydrate groups, such as Sialyl Lewis X or PSGL-1, to L-selectin on neutrophils or E-selectin/P-selectin on endothelial cells. Cytokine stimulation greatly increases expression of endothelial selectins. Activation: Slow rolling allows the leukocytes to sample the chemokines secreted by the inflamed tissue. This activates integrins by inducing a signaling cascade that results in a conformational change in the integrins necessary for binding. Tight adhesion and crawling: Neutrophils become firmly attached to the endothelium via the binding of CD 18 beta 2 integrins (Mac-1 and LFA-1) to intercellular adhesion molecule-1 (ICAM-1) on endothelial cells Transmigration: After crawling to the endothelial cell periphery, neutrophils eventually migrate out of the vasculature by squeezing in between the cells via integrin attachments and adherence to platelet endothelial cell adhesion molecule 1 (PECAM-1). This protein is found primarily at the peripheral intercellular junctions of endothelial cells. Three leukocyte adhesion deficiency (LAD) syndromes have been identified whereby leukocytes cannot leave the vasculature to migrate into tissues under conditions of inflammation. All are rare, autosomal recessive disorders. LAD type 1 results from the absence of CD18. This leads to the inability to synthesize the beta-2 integrins Mac-1 and LFA1, affecting tight adhesion, crawling, and transmigration. The clinical manifestations include recurrent skin infections without pus formation, delayed detachment of the umbilical cord, and poor wound healing. LAD type 2 is a milder condition, with no delay in the separation of the umbilical cord and less severe and fewer infections. LAD type 3 is similar to type 1 and causes severe, recurrent bacterial infections, delayed separation of the umbilical cord, and bleeding complications (due to affected beta-3 integrins on platelets).

A 75-year-old man is brought to the emergency department due to acute-onset profuse vomiting of large amounts of bright red blood. He has had intermittent, dull epigastric pain for the last 3 months. Medical history is significant for chronic obstructive pulmonary disease with multiple exacerbations and lumbar spinal stenosis. The patient has smoked 1 pack of cigarettes daily for 40 years. Blood pressure is 70/40 mm Hg and pulse is 120/min. Examination shows epigastric tenderness. The patient dies an hour after admission despite resuscitation efforts. Autopsy shows a deep peptic ulcer localized proximally on the lesser curvature of the stomach. The ulcer most likely penetrated which of the following arteries?

Left gastric artery The gastroduodenal artery supplies blood to the pylorus and proximal duodenum. Ulcers in the posterior duodenal bulb can erode into the gastroduodenal artery, but a gastric ulcer at the lesser curvature would not involve these arteries.

A 19-year-old man comes to the emergency department after being stabbed in the right arm during a fight. The patient has no chronic medical conditions. Vital signs are within normal limits. Examination of the right upper extremity shows a 4-cm laceration oriented transversely over the volar (flexor) surface of the proximal forearm. The patient is taken to the operating room for surgical repair. While exploring the wound, a nerve that courses between the flexor digitorum superficialis and profundus muscles is found to be transected. Which of the following movements is most likely to be impaired in this patient?

Median nerve is injured. Opposition of the thumb is going to be affected.

A 23-year-old man comes to the emergency department with sudden onset of heart palpitations that started while he was at his desk at work. The patient has no known medical problems and does not use tobacco or illicit drugs. He drinks alcohol occasionally on the weekends. Initial blood pressure is 110/70 mm Hg and pulse is 160/min and regular. Gentle neck massage just below the angle of the right mandible provides immediate improvement of his condition. His blood pressure is now 120/80 mm Hg and pulse is 75/min. Which of the following mechanisms is responsible for improvement of this patient's condition?

Prolonged AV node refractory period. Carotid massage causes increase pressure on carotid sinus which causes increase stretch which causes increase in baroreceptor firing leading to increase in AV node refractory leading to decrease in HR pg 296 first aid

A 35-year-old man comes to the office with progressive fatigue, dyspnea on exertion, and lower extremity edema over the last 2 weeks. Preceding these symptoms, he had an episode of fever, runny nose, and myalgias that resolved after several days. The patient has no other medical history, takes no medications, and has no significant family history. He occasionally drinks alcohol and does not use tobacco. Blood pressure is 112/74 mm Hg, and pulse is 98/min and regular. Physical examination reveals jugular venous distension, bibasilar crackles on lung auscultation, and 2+ pitting edema involving the lower extremities. Which of the following echocardiographic findings is most likely to be seen in this patient? A) concentric left ventricular thickening with abnormal diastolic relaxation B) dialated ventricles with abnormal systolic function C) High systolic pressure gradient across the aortic valve D) Regional wall motion abnormality E)systolic anterior motion of the mitral valve

This patient's presentation with dyspnea, lower extremity edema, jugular venous distension, and pulmonary crackles is consistent with decompensated heart failure. In a young patient who develops heart failure following a symptomatic viral prodrome (eg, fever, sinus congestion, myalgias), dilated cardiomyopathy (DCM) due to viral myocarditis should be suspected. A number of viruses have been implicated in causing DCM including coxsackievirus, adenovirus, and influenza virus. It is believed that an inadequate immune response to the viral infection allows virus to infect and persist inside cardiomyocytes, resulting in damage due to a direct cytotoxic effect or a destructive autoimmune reaction. Depletion of cardiomyocytes with subsequent fibrosis causes weakened myocardial contractility (systolic dysfunction), volume overload, and ventricular dilation (eccentric hypertrophy) as surviving myocytes respond to increased wall stress. Other causes of DCM include genetic disease, pregnancy, infiltrative disease (eg, late manifestation of amyloidosis or hemochromatosis), and drugs/toxins (eg, anthracyclines, chronic alcohol abuse). Educational objective:Dilated cardiomyopathy results from direct damage to cardiomyocytes leading to myocardial contractile dysfunction (systolic dysfunction), volume overload, and ventricular dilation. Viral myocarditis is a common cause of dilated cardiomyopathy and should be suspected in young patients who develop heart failure following a symptomatic viral prodrome.

An 82-year-old man is brought to the emergency department with fatigue and palpitations. The patient was playing chess at the park when his symptoms developed suddenly. He immediately called an ambulance as he has never experienced symptoms like this before. Past medical history is significant for asthma, gout, and hypertension. Blood pressure is 110/70 mm Hg, pulse is 130/min, and rhythm is irregularly irregular. The lungs are clear on auscultation. Intravenous digoxin is administered. Two hours later, the patient appears comfortable; his pulse is 82/min and still irregularly irregular. Which of the following best explains the heart rate lowering effects of the medication?

increased parasymp tone. This patient developed atrial fibrillation with rapid ventricular response (AF with RVR) and was treated with digoxin for ventricular rate control. Although calcium channel blockers and beta blockers are preferred for rate control, digoxin is sometimes used. Digoxin slows the ventricular rate during AF primarily by increasing parasympathetic tone, which leads to inhibition of atrioventricular (AV) nodal conduction. Several mechanisms may explain the parasympathomimetic effects of digoxin. Inhibition of the sodium-potassium ATPase pump in vagal afferent fibers sensitizes arterial baroreceptors (eg, carotid, aortic) and cardiac receptors, augmenting afferent input from the cardiovascular system to the brain. Digoxin also enhances efferent parasympathetic ganglionic transmission, leading to increased vagal output.

A 29-year-old woman comes to the office due to persistent fatigue over the last 4 years. She has also felt unhappy during this period, ever since being let go from her previous job. The patient describes her fatigue as "having little energy to do things." When asked what she enjoys, she replies that "everything in life is a chore" and that she feels hopeless that her life will improve. The patient has no suicidal thoughts, problems with concentration, or changes in appetite or sleeping patterns. She used marijuana as a teenager and drinks 1 or 2 glasses of wine on weekends. Detailed workup, including urine toxicology screen, is negative. Which of the following is the most likely diagnosis?

persistent dysthymic depression This patient's chronic depression, fatigue, and hopelessness are consistent with a diagnosis of persistent depressive disorder. Diagnosis requires depressed mood more days than not for at least 2 years (1 year in children) and at least 2 other depressive symptoms (Table). In DSM-5, persistent depressive disorder includes both "pure dysthymic syndrome" and chronic major depression or dysthymia with concurrent or intermittent major depressive episodes. This patient would be diagnosed with "pure dysthymic syndrome" as she has never met the criteria for a major depressive episode. Diagnosis of persistent depressive disorder requires ruling out medical and substance-induced etiologies and differentiation from other psychiatric disorders. Persistent depressive disorder is treated with antidepressants, psychotherapy, or a combination of these.

A researcher is studying the effect of various manipulations on kidney blood flow and glomerular filtration. Which of the following is most likely to both decrease renal plasma flow and increase the filtration fraction? A) constriction of efferent arteriole b) constriction of affarent arteriole C) bladder neck obstruction D) dialation of the efferent arteriole

A check out changes in glomerular dynamixs in first aid

A 76-year-old hospitalized woman is evaluated for persistent fevers. Four days ago, she was admitted to the hospital for sepsis, for which she has received broad-spectrum empiric intravenous antibiotic therapy. Her temperature is 38.9 C (102 F), blood pressure is 110/80 mm Hg, pulse is 98/min, and respirations are 18/min. Cardiac examination reveals a new diastolic murmur. Multiple blood cultures drawn at different times grow Enterococcus. Echocardiogram reveals mitral valve vegetation suggestive of endocarditis. An intravenous antibiotic is added to the patient's treatment regimen for synergy. Several days later, she develops tinnitus and hearing loss attributed to this antibiotic. The additional antibiotic most directly affects which of the following processes? A) tRNA CHARGING b) mRNA genetic code reading

B AGs irreversibly bind to bacterial 30S but not eukaryotic 40S; this causes genetic code misreading and bacterial protein synthesis inhibition. AGs also appear to impact translocation, whereby an aminoacyl-tRNA is shifted from the ribosomal A site (after initial binding) to the P site (after AA incorporation into the peptide chain during elongation) to the E site (after AA cleaving from its tRNA). The initial N-formylmethionine-tRNA notably binds at the P site.

A 65-year-old man comes to the office due to pain, redness, and swelling in his right calf. The patient is diagnosed with cellulitis and is started on clindamycin. A few days after starting treatment, he develops watery diarrhea and abdominal cramps. The patient is hospitalized, and complete blood count reveals leukocytosis. The toxin responsible for his current condition most directly impairs which of the following components of intestinal mucosal cells? A) cell membrane integrity B) cytoskeleton integrity

B These strains produce toxin A and toxin B, which act synergistically, although toxin B is significantly more virulent. The toxins bind specific receptors on intestinal mucosal cells and are internalized, allowing them to inactivate Rho-regulatory proteins involved in actin cytoskeletal structure maintenance. The result is loss of cytoskeleton integrity, leading to cell rounding/retraction, disruption of intercellular tight junctions, and increased paracellular intestinal fluid secretion (eg, watery diarrhea). Both toxins also have inflammatory effects (eg, neutrophil recruitment) and can induce apoptosis, resulting in pain and pseudomembrane formation.

A 13-year-old boy is brought to the clinic due to progressive difficulty breathing, hoarseness, and dysphagia. The symptoms began insidiously 6 months ago but have rapidly become worse in the last 4 weeks. The patient has no associated skin rash, pharyngeal pain, fever, weight loss, or loss of appetite. Medical history is unremarkable, and the patient takes no medications. He is up to date on scheduled vaccinations. The patient's parents and siblings have no history of significant health problems. Vital signs are normal, and he is at 70th percentile for height and weight. Oropharyngeal examination reveals a large, red lingual mass. The patient subsequently undergoes excision of the mass; histopathologic preparation of the recovered tissue is shown in the image below. Failure of which of the following embryologic processes is most likely responsible for this patient's lesion? A) apoptosis B) migration C) Fusion D) Differentiation E) Proliferation

B This patient's pathologic specimen shows normal thyroid follicles containing colloid, consistent with a lingual thyroid. The thyroid gland is formed from an outpouching (evagination) of the pharyngeal epithelium and subsequently descends to the lower neck anterior to the upper trachea and larynx. The lowest part of the evagination forms the thyroid gland and the remaining portion forms the thyroglossal duct, which extends from the foramen cecum on the dorsal surface of the tongue to the superior border of the thyroid isthmus. Due to failure of migration, the thyroid can reside anywhere along the thyroglossal duct's usual path, including the tongue (lingual thyroid). Enlargement of a lingual thyroid can lead to obstructive symptoms (eg, dysphagia, dysphonia, dyspnea), typically during times of heightened thyroid stimulation (eg, puberty, pregnancy). Sometimes, this lingual thyroid is the only thyroid tissue in the body, so significant hypothyroidism occurs if it is removed.

A 2-year-old boy is evaluated for easy bruising. His parents report that he develops marked bruising and open wounds following minor trauma. The skin is difficult to suture due to its extreme fragility. Physical examination reveals hyperextensible skin, multiple ecchymoses over the forearms and pretibial regions, and an umbilical hernia. A skin biopsy is performed, and histochemical evaluation of the biopsy reveals a defect in extracellular processing of collagen. Which of the following steps of collagen synthesis is most likely impaired in this patient? A) glycosylation of hydroxylysine residues B) interchain C terminal disulfide bond formation C) N terminal propeptide removal D) proline residues hydroxylation E) triple helix formation

C Ehlers-Danlos syndrome is a group of rare hereditary disorders characterized by defective collagen synthesis. It can be caused by procollagen peptidase deficiency, which results in impaired cleavage of terminal propeptides in the extracellular space. Patients often have joint laxity, hyperextensible skin, and tissue fragility due to the formation of soluble collagen that does not properly crosslink.

A 34-year-old industry worker experiences rapid-onset shortness of breath, dizziness, palpitations, and flushed skin after accidental exposure to chemical fumes. Examination by an occupational physician shows that the patient is tachypneic and has a reddish discoloration of the skin without cyanosis. The patient is immediately instructed to inhale amyl nitrite from the safety kit. The success of this antidote is dependent upon its ability to convert hemoglobin into a form with increased affinity for which of the following substances? A) Carbon dioxide B) iron C) cyanide D) 2, 3 bisphosphoglycerate

C Recall nitrites oxidize hb by converting Fe2+ to Fe 3+ turns out that Fe3+ has a high affinity for cyanide which basically sequesters and is less toxic

A 6-year-old girl is brought to the emergency department after falling from a chair onto her outstretched right arm. Physical examination shows swelling of the right elbow and tenderness to palpation over the distal humerus. Radial pulses are full and symmetric bilaterally. The patient is unable to cooperate with neurologic examination of the right hand due to significant pain. A peripheral intravenous line is placed, and analgesics are administered for pain control. X-rays show a supracondylar humeral fracture with anterolateral displacement of the proximal fracture fragment. Which of the following structures is most likely to be injured in this patient? A) ulnar nerve B) median nerve C) radial nerve D) axillary

C radial because it says anterolateral displacement if it had said anteromedial then it would be median Supracondylar humeral fractures are common pediatric elbow fractures that characteristically occur after hyperextension injuries (eg, falling onto an outstretched arm). These fractures carry a significant risk of neurovascular compromise, especially if the fragments are displaced. The brachial artery, median nerve, and radial nerve all run anterior to the elbow, with the brachial artery and median nerve over the medial epicondyle and the radial nerve over the lateral epicondyle. The median nerve and brachial artery are frequently injured in supracondylar fractures due to anteromedial displacement of the proximal fracture fragment (Choice C). However, this patient has intact radial pulses bilaterally (ie, sparing of the brachial artery) and anterolateral displacement of the proximal fracture fragment, making radial nerve injury more likely. Injury to the radial nerve at the elbow can result in wrist drop due to denervation of hand/finger extensor muscles, and sensory loss over the posterior forearm/dorsolateral hand.

A 45-year-old man is brought to the emergency department due to new-onset seizure. His family witnessed an episode of blank staring followed by a generalized tonic-clonic convulsion with tongue biting and urinary incontinence. The patient has a past medical history of seasonal asthma and takes no prescription medications. He does not use alcohol or illicit drugs. Vital signs and neurologic examination are normal. Noncontrast head CT and brain MRI are consistent with a diagnosis of cavernous hemangioma. This patient is at greatest risk of developing which of the following complications? A) subarachnoid hemorrhage B)Epidural hematoma C) intracerebral hemorrhage

Cavernous hemangiomas are vascular malformations (sporadic or familial) that most commonly occur within the brain parenchyma above the cerebellar tentorium. They consist of clusters of dilated, thin-walled capillaries with little or no intervening nervous tissue. Patients with cavernous hemangiomas often have seizures and may develop progressive neurologic deficits due to mass effect with interruption of the cerebral microcirculation. These lesions also have a tendency to bleed, which can contribute to neurologic symptoms (eg, due to subcortical/cortical tissue irritation) and appear as an irregular hyperdense/bright mass on head CT.

A study is conducted to standardize laboratory equipment in a hospital network. In one hospital, 2 healthy volunteers undergo testing. Both are found to have a serum creatinine level of 1.1 mg/dL. Glomerular filtration rate is estimated using the same equation and reveals values of 118 mL/min in one volunteer and 70 mL/min in the other. A difference in which of the following parameters best explains the observed laboratory findings in these volunteers? A) BMR b) dietary purine intake C) renal tubular reabsorption capacity D) skeletal muscle mass

Creatinine is a waste product generated by the breakdown of creatine in the muscles. It is released from muscle at a relatively constant rate, is neither metabolized nor reabsorbed by the kidneys, and is easily measured; therefore, it can be used to estimate the glomerular filtration rate (GFR). However, creatinine has several limitations in the estimation of GFR. Because its formation is due largely to muscle metabolism, differences in skeletal muscle mass (eg, higher in body builders and lower in elderly patients, those with amputations) affect the amount of creatinine synthesized. Alterations in dietary intake can also raise (eg, creatine supplements, high-meat diet) or lower (eg, low-protein vegetarian diet) creatinine levels. Therefore, patients with lower muscle mass or reduced dietary meat intake may have significantly lower GFRs for any given creatinine level. Another potential source of error is the active secretion of creatinine by the proximal tubules; if uncorrected, this results in a slight overestimation of GFR (~10%-20%).

A researcher is conducting a retrospective study on breast cancer recurrence. Records of a number of patients with hormone receptor-positive, early-stage breast cancer who received adjuvant therapy with tamoxifen were evaluated. Various clinical, demographic, and drug concentration data were analyzed. A comparison of patients who had disease recurrence with those who remained cancer free showed that some of the relapsed patients had lower serum concentrations of endoxifen and 4-hydroxytamoxifen, the active metabolites of tamoxifen. Which of the following is the most likely cause of the drug's ineffectiveness in this subset of patients?

Cytochrome P450 (CYP) enzymes are a group of heme-containing proteins that are responsible for the majority of drug metabolism, which occurs predominately in the liver. Various CYP subtypes exist, with CYP3A, CYP2D, and CYP2C as the most active subtypes involved in drug metabolism. These enzymes generally function to deactivate drugs and facilitate excretion from the body by improving water solubility. However, they also metabolize certain compounds to their active forms. Polymorphisms may occur in the genes coding for these enzymes, altering their expression or activity. Three important phenotypes exist: poor, intermediate, and rapid metabolizer. Identifying these variations on an individual basis provides a framework for optimizing therapy, predicting treatment efficacy, and minimizing toxicity. Tamoxifen, a selective estrogen receptor modulator used in the treatment of estrogen receptor-positive breast cancer, is a prodrug metabolized by CYP2D to its active metabolite, endoxifen. Patients with genetic polymorphisms resulting in poor CYP2D activity are exposed to decreased levels of the active metabolite and have a higher risk of disease relapse.

A 78-year-old male nursing home resident is brought to the physician because of abdominal pain and discomfort. He has a history of advanced dementia and is only partially able to verbalize his symptoms. He has had intermittent abdominal discomfort for years, with no other related symptoms. He denies diarrhea and rectal bleeding but has not had a bowel movement in approximately 5 days. The patient is largely bed-bound, and only has minimal activity with the help of nurses and physical therapists. Past regular colonoscopies have shown only benign lesions. His other medical problems include dementia, coronary artery disease, hypertension, spinal stenosis, and osteoarthritis of his hips and knees. Abdominal examination does not show tenderness, masses, or hepatosplenomegaly, although fullness is appreciated. The remainder of the examination shows no abnormalities. Polyethylene glycol is administered and produces a bowel movement within 24 hours. The mechanism of action of polyethylene glycol in this patient is most similar to the pathophysiology of which of the following disorders? A) IBS B)Celiac disease C)Crohns D)Lactase def E) Rectal prolapse

D Polyethylene glycol is an osmotic laxative. Diarrhea associated with lactase deficiency is also osmotic and occurs due to accumulation of nonabsorbable lactose in the intestinal lumen. Magnesium hydroxide (and other magnesium-containing compounds, such as magnesium citrate) is another osmotic laxative that is often used, although its efficacy is questionable and there is not enough evidence to support its widespread use.

WHY DOES HEP D need hep b?

It needs hep B surface antigen to coat itself. The internal polypeptide assembly of HDV is designated hepatitis D antigen (HDAg).Associated with this antigen is a very short, circular molecule of single-stranded RNA. HDAg is considered replication defective as it must be coated by the external coat hepatitis B surface antigen (HBsAg) of HBV to penetrate the hepatocyte. HDV infection can arise either as an acute coinfection with HBV or as a superinfection of a chronic HBV carrier.

A 24-year-old previously healthy woman comes to the hospital with a 3-day history of fever, dyspnea, and productive cough of yellow sputum. Her temperature is 38.8 C (102 F), blood pressure is 110/66 mm Hg, and pulse is 110/min. She has bronchial breath sounds and crackles in the right lower lung field. Laboratory studies are as follows: Hemoglobin13 g/dL Platelets350,000/mm3 Leukocytes54,000/mm3 Neutrophils65% Band form10% Myelocyte3% Metamyelocyte1% Lymphocytes15% The leukocyte alkaline phosphatase test score is elevated. Which of the following is the most likely additional finding on this patient's blood smear?

This patient most likely has pneumonia with sepsis and a leukemoid reaction, defined as a benign leukocytosis (>50,000/mm3) that occurs in response to an underlying condition such as severe infection/hemorrhage, solid tumors, or acute hemolysis. The bone marrow can be normal or hypercellular and responds with increased bands and early mature neutrophil precursors (eg, myelocytes), in contrast to the increased immature cells (eg, promyelocytes, myeloblasts) seen in acute leukemia. Serum leukocyte alkaline phosphatase can be normal or increased in leukemoid reaction, but it is usually low in chronic myelogenous leukemia. The peripheral smear can show Döhle bodies, which are light blue (basophilic) peripheral granules in neutrophils. The blue color is likely due to ribosomes bound with rough endoplasmic reticulum. Döhle bodies are commonly seen in toxic systemic illness but can also occur with burns or myelodysplasia. Other findings of systemic inflammation or infection include increased bands (left shift), toxic granulation, and cytoplasmic vacuoles.

The patient is tachypneic and unable to speak in full sentences. Examination reveals prolonged expiration and prominent bilateral wheezing. Heart sounds are normal. Chest imaging shows a normal-sized heart and hyperinflated lungs with a flattened diaphragm. Bedside echocardiogram reveals no intrapericardial fluid accumulation or pericardial thickening. Which of the following physiologic changes is most likely to provide immediate relief in this patient?

This patient's presentation with dyspnea, tachypnea, prolonged expiration, and bilateral wheezing is suggestive of acute exacerbation of obstructive pulmonary disease (asthma or COPD). Acute obstructive pulmonary exacerbations are primarily treated with beta-adrenergic agonists, which produce relaxation of bronchial smooth muscle by stimulating the beta-2-adrenergic receptor. This receptor is a GS protein-coupled receptor that activates adenylyl cyclase and increases intracellular cAMP concentrations. Asthma and COPD exacerbation are the most frequent causes of pulsus paradoxus in the absence of significant pericardial disease (pericardial effusion or constrictive pericarditis). There is a small normal variation in intrathoracic pressure with respiration, which drops 2-5 mm Hg below the atmospheric pressure during inspiration. In patients with severe asthma or COPD exacerbation, this drop in intrathoracic pressure becomes greatly exaggerated and is transmitted to extrathoracic structures. This leads to an excessive drop in blood pressure with inspiration that is detected as pulsus paradoxus.

A 34-year-old woman comes to the office for evaluation of recurrent transient pulmonary infiltrates. The patient has a history of bronchial asthma and has had several exacerbations over the past few years, particularly during the winter months. She is currently asymptomatic. She has no other medical problems and has never traveled outside the United States. Her medications include albuterol as needed and medium-dose inhaled glucocorticoids. Temperature is 37.1 C (98.8 F). Physical examination is unremarkable. Complete blood count shows eosinophilia. A chest CT scan reveals proximal bronchiectasis. This patient's condition is most likely related to colonization with which of the following?

allergic brochopulmonary Aspergillosus Allergic bronchopulmonary aspergillosis (ABPA) due to Aspergillus fumigatus may complicate asthma. ABPA can result in transient recurrent pulmonary infiltrates and eventual proximal bronchiectasis. Microbiology Subject Pulmonary & Critical Care System Aspergillosis

A 56-year-old man comes to the emergency department due to palpitations and lightheadedness over the last 2 hours. He has a history of hypertension but no other cardiac conditions and never before had similar symptoms. The patient drinks 1 to 2 alcoholic beverages daily but does not smoke cigarettes or use illicit drugs. Blood pressure is 132/76 mm Hg and pulse is 116/min and irregular. Cardiopulmonary examination is normal, except for tachycardia. Resting ECG shows atrial fibrillation. The patient undergoes transesophageal echocardiography, which reveals normal cardiac anatomy and no intracardiac thrombi. Electrical cardioversion is performed. After the intervention, sinus rhythm is restored, and amiodarone therapy is begun to maintain normal sinus rhythm. Which of the following cardiac electrophysiologic changes are most likely to occur in this patient due to the medication? What happens to sinus rate, PR interval, QRS and T intervals

amiodarone causes sinus rate to decrease and it increases everything else Amiodarone primarily functions as a class III antiarrhythmic, inhibiting the delayed rectifier potassium current to slow ventricular repolarization and prolong the QT interval. It also inhibits fast sodium channels (class I effect) to slow ventricular depolarization and prolong QRS complex duration. Beta blockade (class II effect) and inhibition of slow L-type calcium channels (class IV effect) slow conduction in the sinus node and atrioventricular node causing decreased sinus rate and a prolonged PR interval.

The prevalence of a chronic disease that affects mostly older individuals has been constant in a population for the last 15 years. As a result of worsening economic conditions that have no effect on the disease itself, many young healthy individuals emigrate from the population. Which of the following is the most likely effect of the emigration on the estimate of disease prevalence over the next few years? A) prevalance will decrease B) prevalance will increase C) remain the same D) not enough info

b

A 52-year-old man comes to the office with concerns about a "heart problem." Three years ago, the patient was told during a routine physical examination that he had a heart murmur. Currently, he has no specific complaints, but he mentions that he has become tired more easily in the past year. He has no chest pain or lower extremity swelling or significant medical history. The patient was healthy as a child and received all his childhood vaccinations. Blood pressure is 145/90 mm Hg and pulse is 80/min and regular. Physical examination reveals a holosystolic murmur best heard at the apex of the heart that radiates to the axilla. The lungs are clear on auscultation and abdominal examination is unremarkable. Which of the following is the best indicator of the severity of this patient's valve disease? A) Presence of mid systolic click B) audible S3 C) Audible s4 D) decrease in distance between s2 an opening snap

b A holosystolic murmur at the apex with radiation to the axilla is consistent with mitral regurgitation (MR). The murmur is generated by regurgitant blood flow from the left ventricle back to the left atrium during systole. This results in elevated pressure and blood volume in the left atrium, which increases the amount of blood reentering the left ventricle during diastole; an audible S3 gallop occurs when the left ventricle is unable to accommodate the excess blood flow. An S3 is generated by the sudden cessation of blood flow into the left ventricle during the passive filling phase of diastole. A higher volume of blood flow or a more dilated left ventricle is more likely to produce an S3, and although an S3 can be a normal finding in healthy young adults, it is usually a pathologic finding in older adults that is classically associated with heart failure. In patients with MR, an S3 is the best indicator of severe MR with left-sided volume overload; the absence of an S3 can be used to exclude severe chronic MR.

A 20-year-old man suddenly collapses while walking on a college campus. He is found to be unresponsive and pulseless by a bystander. Despite efforts by emergency medical personnel, the man is unable to be resuscitated. His friends say he appeared fatigued and described palpitations and dyspnea with exertion in the weeks leading up to the event. He had no known medical history and did not use tobacco, alcohol, or illicit drugs. The man's 3 younger siblings are healthy. His parents confirm a history of "early heart disease" in a paternal grandfather and request an autopsy and genetic testing for further evaluation. Genetic studies reveal a defect in a TTN gene exon resulting in a truncated titin protein. Autopsy findings are most likely to show which of the following phenotypes? A) hypertrophic cardiomyopathy B) Dialated cardiomyopathy C) Restrictive cardiomyopathy

b Familial DCM can result from a variety of genetic mutations affecting either sarcomere (ie, the contractile apparatus) or nonsarcomere proteins. Truncating mutations (usually nonsense mutations) affecting the TTN gene, which encodes for the sarcomere protein titin, are the most common cause of familial DCM. Titin is an elastic protein that anchors the beta-myosin heavy chain to the Z-discs and likely contributes to passive myocardial tension; absence of complete titin proteins leads to myocardial dysfunction.

A 19-year-old man comes to the office for evaluation of a hand injury. He says, "I was just minding my own business at work when this dog bit me." The patient recently started operating a dog-fighting club and adds that his "entrepreneurial skills" have made him "richer than a doctor." He has been fired from multiple jobs, most recently while working as a car salesman, due to getting into arguments with coworkers and being repeatedly late to shifts. The patient says, "I was only late a few times. Haven't you ever been late before, doc?" During middle school, he was recurrently truant and spent time at a juvenile detention center for tampering with his teacher's car brakes after failing a class. Which of the following is the most likely diagnosis? A) Conduct disorder B) antisocial C)narcissist Personality D) dependent

b This patient's operation of an illegal dog-fighting ring, unstable employment history, irritability with coworkers, and repeated tardiness to work are most consistent with a diagnosis of antisocial personality disorder (ASPD). This disorder is characterized by a pattern of disregard for and violation of the rights of others, beginning by early adolescence and persisting into adulthood (age ≥18 required for diagnosis). An essential feature for diagnosis of ASPD is evidence of conduct disorder prior to age 15 (eg, this patient's history of truancy and vindictive behavior toward his teacher). Individuals with ASPD display consistent irresponsibility along with impulsivity and irritability, which may result in physical aggression. Engagement in criminal activities (eg, theft, illegal occupations) and lack of remorse for transgressions are typical in this disorder.

A 4-week-old infant is evaluated for frequent episodes of hypoglycemia. The patient is found to have elevated insulin levels during periods of fasting. Several members of his family have a similar condition. Genetic testing reveals a mutation involving potassium channels expressed in pancreatic beta cells. The mutated channels have a higher sensitivity to the substance that normally modulates their activity, causing increased beta cell secretory activity. Which of the following substances normally binds to and regulates the ion channels that are responsible for this patient's hypoglycemia? A) glucose B) ATP c) Citrate D) fructose 6 phosphate

b remember first you eat food gets converted to glucose, it goes into glycolysis, TCA cycle and then you make ATP this causes increase in ATP/ADP ration leading to closure of ATP dependent K channels causing insulin release pancreatic beta cells secrete insulin in response to elevated glucose levels. Glucose enters beta cells via glucose transporter 2 (GLUT-2) and is metabolized via glycolysis and the citric acid cycle to generate ATP. ATP then binds to the regulatory subunit of the ATP-sensitive potassium channel (KATP channel). KATP channels are normally open at rest and maintain membrane polarization by allowing outward movement of potassium from the beta cells; however, on binding ATP, the KATP channels close. Increased intracellular ATP therefore leads to decreased potassium efflux and membrane depolarization. This triggers opening of voltage-dependent calcium channels, increased intracellular calcium levels, and subsequent insulin release. Mutations in the KATP channel causing increased affinity for ATP result in fewer open channels and depolarization at lower glucose concentrations. This causes continued secretion of insulin despite falling blood glucose levels, leading to congenital hypoglycemia. Conversely, mutations of the KATP channels that decrease ATP affinity prevent appropriate depolarization in response to rising glucose levels, causing a familial form of neonatal diabetes mellitus. These patients can be treated successfully with sulfonylureas, which bind to KATP channels and cause closure independent of ATP.

A 22-year-old woman is hospitalized due to suicidal ideation. The patient describes feelings of intense sadness, emptiness, and despair for the past week following the breakup of her month-long romantic relationship. For the past week, she has had difficulty falling and staying asleep but has had no change in appetite, energy level, or concentration. She feels betrayed by her boyfriend and is furious at her parents for turning against her. Since the breakup, the patient has been going to bars, engaging in unsafe sex, and drinking excessively. She describes having intense mood swings since adolescence and has a history of 2 suicide attempts involving an overdose of sleeping pills. Which of the following is the most likely diagnosis?

borderline personality disorder This patient's depression, suicidal ideation, and impulsivity in the setting of feeling rejected are characteristic of borderline personality disorder (BPD). Individuals with this disorder exhibit a persistent pattern of unstable and intense relationships beginning in adolescence or early adulthood. They experience marked mood reactivity to interpersonal stresses and frequently alternate between extremes of idealizing and devaluing others (defense mechanism of splitting). Difficulty controlling anger, impulsivity (eg, substance use, unsafe sex), recurrent suicidality, and self-mutilating behavior (eg, cutting, burning) are common. Individuals with BPD frequently seek clinical attention due to suicidal behavior or threats during an interpersonal crisis and comprise approximately 20% of psychiatric inpatients

A 34-year-old man develops excessive thirst and frequent urination while being hospitalized for multiple injuries following a motorcycle collision. His urinary osmolality after 4 hours of water restriction is 250 mOsm/L and increases to 700 mOsm/L after administration of antidiuretic hormone. The patient is started on an appropriate medication, and his symptoms promptly resolve. Over the next 2 years, multiple attempts are made to wean the patient off the medication, but his symptoms would always return, necessitating the resumption of treatment. The permanence of this patient's condition is best explained by traumatic damage to which of the following structures? A) anterior pituitary B) post pituitary C)hypothalamic nuclei

c ADH is synthesized by magnocellular neurosecretory cells in the paraventricular and supraoptic nuclei of the hypothalamus. These neurons project axons to the posterior pituitary where axon terminals store large quantities of ADH for later release into the circulation. Injury to the hypothalamus or posterior pituitary (eg, head trauma, transsphenoidal neurosurgery, suprasellar masses) can result in central DI. Damage to the more distal portions of the hypothalamic-hypophyseal tract (ie, below the infundibulum) typically causes transient DI because the cell bodies of the magnocellular neurons remain intact. However, hypothalamic injury results in death of the magnocellular neurons, causing permanent central DI.

A 31-year-old man comes to the office for a routine checkup. He has no significant medical problems and does not take any medications. The patient works as a fitness trainer and lifts weights recreationally. He has been consuming carbohydrate-rich food prior to his weightlifting sessions and claims that it increases muscle strength. A literature review shows that the rate of glycogenolysis within myocytes increases several hundredfold during active skeletal muscle contraction. Which of the following substances is most likely responsible for increasing the reaction rate during active contraction? A) ATP B)Ca2+ C)CAMP D) lactate

ca2+ Glycogen is broken down by the enzyme glycogen phosphorylase, which is regulated through phosphorylation (active state) and dephosphorylation (inactive state). Phosphorylase kinase (PK) is the enzyme responsible for the phosphorylation of glycogen phosphorylase, whereas phosphoprotein phosphatase catalyzes its dephosphorylation. PK is regulated differently in liver than in muscles. Glycogen stored in the liver is used to maintain blood glucose levels during the fasting state, whereas glycogen in the muscles is used to provide energy for muscle contraction. In the liver, PK is activated primarily through the binding of epinephrine and glucagon to Gs protein-coupled receptors, which increases cAMP concentrations and causes phosphorylation of PK (via protein kinase A). Skeletal muscle lacks glucagon receptors, but muscle PK can still be phosphorylated in response to an epinephrine-induced increase in cAMP concentrations. However, increased intracellular calcium is a more powerful activator of muscle PK. Release of sarcoplasmic calcium stores following neuromuscular acetylcholine stimulation allows for synchronization of skeletal muscle contraction and glycogen breakdown, providing the energy necessary for anaerobic muscle contraction.

67-year-old man comes to the emergency department due to progressive shortness of breath and chest tightness. He has had no lightheadedness or syncope. The patient takes lisinopril for hypertension and metformin for type 2 diabetes mellitus. He has smoked a pack of cigarettes daily for the last 40 years. The blood pressure cuff is inflated to 140 mm Hg and the pressure is released very slowly. At 120 mm Hg, intermittent Korotkoff sounds are heard only during expiration. At 100 mm Hg, Korotkoff sounds are heard throughout the respiratory cycle. This physical examination finding can be seen in which of the following conditions? A) Aortic valve disease B)Mitral valve disease C) Myocardial ischemia D) Pericardial disease

d Systemic arterial pressure normally falls by <10 mm Hg during normal inspiration. Pulsus paradoxus refers to an exaggerated drop (>10 mm Hg) in systolic blood pressure during inspiration. It is detected by inflating a blood pressure cuff above systolic pressure and gradually deflating it. The difference between the systolic pressure at which Korotkoff sounds first become audible during expiration and the pressure at which they are heard throughout all phases of respiration quantifies pulsus paradoxus (20 mm Hg in this patient). Inspiration causes an increase in systemic venous return, resulting in increased right heart volumes. Under normal conditions, this results in expansion of the right ventricle into the pericardial space with little impact on the left side of the heart. However, in conditions that impair expansion into the pericardial space (eg, acute cardiac tamponade), the increased right ventricular volume occurring with inspiration leads to bowing of the interventricular septum toward the left ventricle. This leads to a decrease in left ventricular (LV) end-diastolic volume and stroke volume, with a resultant decrease in systolic pressure during inspiration. Pulsusparadoxus—in amplitude of systolic BP by>10mm Hg during inspiration. Seen in constrictive Pericarditis, obstructive pulmonary disease (eg, Croup, OSA Asthma, COPD),cardiacTamponade(PeaCOAT)

A 29-year-old previously healthy man comes to the hospital with a right leg injury after a cycling accident. He reports pain in the lower leg and inability to bear weight. X-ray reveals right tibial and fibular shaft fractures. The patient undergoes closed reduction and immobilization of the fractures. Several hours later, he has increasingly severe pain in his right leg. Physical examination shows increased tension within the anterior compartment. Which of the following structures is most likely to be compromised by this patient's acute complication?

deep peroneal nerve The deep fascia of the leg, the fascial intermuscular septae, and the interosseous membrane divide the leg into 4 compartments (anterior, lateral, and superficial and deep posterior). The fascia is inelastic and limits outward expansion of the contracting muscles. If the pressure in a compartment increases, the circulation within it can become compromised, leading to severe pain and eventual tissue ischemia. This complication, acute compartment syndrome (ACS), can be due to long-bone fractures, crush injury, thermal injury, or nontraumatic causes (eg, bleeding disorders, vascular disease). Early diagnosis and surgical decompression (fasciotomy) of the involved compartments are necessary to prevent permanent and disabling injury (eg, myonecrosis, nerve damage, amputation). The most common site for ACS is the anterior compartment of the leg, which includes the foot extensor muscles, anterior tibial artery, and the deep peroneal (fibular) nerve. Injury to the deep peroneal nerve causes decreased sensation between the first and second toes, decreased dorsiflexion of the foot, foot drop, and claw foot.

A 21-year-old man is brought to the emergency department in a stuporous condition. He returned from a family visit to Mexico 2 weeks ago. One week ago, the patient began developing high-grade fever, headaches, severe myalgias, and joint pains. He had similar symptoms following a trip to Mexico 5 years ago, but at that time the symptoms did not progress and instead resolved spontaneously within a few days. Temperature is 38.3 C (101 F), blood pressure is 80/50 mm Hg, and pulse is 128/min. Examination shows a diffuse maculopapular rash, multiple purpuric lesions, and hepatomegaly. Laboratory studies show marked thrombocytopenia, leukopenia, and elevated liver aminotransferases. Which of the following is the most likely mechanism for this patient's current severe illness?

different viral serotype

A 32-year-old woman comes to the emergency department due to severe headache, nausea, and vomiting for the past several hours. The headache is pounding and localized to the left side. She also describes bright spots in her vision prior to headache onset. The patient has had similar episodes occasionally since adolescence. Physical examination shows normal pupils and fundi, but the patient has discomfort to bright light. There is no facial droop, extremity weakness, or sensory loss. The patient is treated with metoclopramide and diphenhydramine. Which of the following is the most likely reason diphenhydramine was used in this patient?

diphenhydramine will block muscarnic receptor and prevent EXTRAPYRAMIDAL EFFECT. Dopamine receptor blockade in the basal ganglia can also cause excess cholinergicactivity that manifests as significant extrapyramidal symptoms, such as acute dystonic reaction (eg, torticollis, oculogyric crisis). Therefore, diphenhydramine can be co-administered with metoclopramide or prochlorperazine to prevent such reactions due to its anticholinergic activity.

A 34-year-old woman comes to the hospital with a 4-day history of abdominal cramps, nausea, and watery diarrhea. Today she developed dizziness on standing. Her child has had similar symptoms recently. The patient has no prior medical conditions and takes no medications on a regular basis. Blood pressure is 124/82 mm Hg while supine and 100/70 on standing; pulse is 98/min. Examination shows dry mucous membranes. The abdomen is soft and nontender. Laboratory results are as follows: Serum chemistry Sodium144 mEq/L Blood urea nitrogen50 mg/dL Creatinine1.8 mg/dL Urinalysis Protein: negative Red blood cells 0/hpf White blood cells 0-1/hpf Microscopy few hyaline casts Urine sodium 8 mEq/L Which of the following changes are most likely to be seen in this patient? vasopressin, endothelin, angiotensin , norepi

everything increased. This patient with gastroenteritis has evidence of hypovolemia (dry mucous membranes, orthostatic dizziness/hypotension) and acute kidney injury. Decreased extracellular fluid volume stimulates compensatory mechanisms directed at maintaining systemic blood pressure and tissue oxygenation. This is largely driven by the sympathetic nervous system and the kidneys: Activation of the renin-angiotensin-aldosterone system (RAAS) leads to elevated levels of angiotensin II (ATII), a potent vasoconstrictor that stimulates the release of aldosterone and endothelin 1. These increase sodium and water reabsorption and systemic vascular resistance to help maintain blood pressure. Vasopressin (antidiuretic hormone) is released by the posterior pituitary in response to increased serum osmolarity and decreased systemic pressure; it increases urea and free water reabsorption by the renal collecting duct. Increased sympathetic activity results in release of circulating norepinephrine and other catecholamines, which increases systemic vasoconstriction, renal sodium and water reabsorption, and heart rate. These neurohumoral mechanisms promote volume expansion and increase blood pressure, helping to maintain tissue perfusion. Laboratory studies characteristically reflect sodium, water, and urea reabsorption by the kidney, including low urine sodium (<20 mEq/L), low fractional excretion of sodium (<1%), and elevated blood urea nitrogen to creatinine ratio (>20:1). Urinary sediment is typically bland or may show hyaline casts (which suggest increased urine concentration).

A 21-day-old boy is brought to the office by his mother because of a palpable bulge in the child's neck. He continues to feed well but appears comfortable only when held with his body sideways under the breast. He is at the 50th percentile for height, weight, and head circumference. The child favors looking toward the right and cries when his head is turned to the left. There is a firm mass on the left side of his neck that does not move when the child swallows. The remainder of the examination is unremarkable. Which of the following conditions was most likely present prenatally?

intrauterine malposition This infant has congenital torticollis, which typically develops by 2 to 4 weeks of age. It is most commonly caused by birth trauma (eg, breech delivery) or malposition of the head in utero (eg, due to fetal macrosomia or oligohydramnios), both of which can result in sternocleidomastoid muscle (SCM) injury and fibrosis. Children with congenital torticollis may have additional musculoskeletal anomalies, including hip dysplasia, metatarsus adductus (ie, adduction of the forefoot), and talipes equinovarus (ie, clubfoot). The diagnosis of congenital torticollis is made clinically. On physical examination, the head is tilted toward the affected side with the chin pointed away from the contracture. A soft-tissue mass may be palpable in the inferior one-third of the affected SCM. Plagiocephaly and facial asymmetry may be seen in severe cases. Most cases resolve with conservative therapy and stretching exercises.

Researchers studying fungal pathogens isolated an ergosterol biosynthetic gene by polymerase chain reaction. Using homologous recombination, they then developed a new strain of Candida with a mutated allele for that gene. The mutant strain was cultured in the presence of several different antifungal agents. Compared to isolates from the wild-type strain, isolates from the mutant Candida strain exhibited enhanced growth in the presence of drug X. Further investigation revealed that the mechanism of resistance to drug X was a decrease in ergosterol incorporation into the cell membrane. Based on this data, drug X is likely to be most closely related to which of the following antifungal agents?

nystatin The main classes of antifungal medications currently in use include: Polyenes (eg, amphotericin B, nystatin): Bind to ergosterol molecules in fungal cell membranes, creating pores and causing cell lysis Triazoles (eg, ketoconazole, fluconazole, itraconazole, voriconazole): Inhibit the synthesis of ergosterol Echinocandins (eg, caspofungin, micafungin): Inhibit the synthesis of glucan, a component of the fungal cell wall Pyrimidines: Flucytosine, the only agent in this class of antifungals, is converted to 5-fluorouracil within the fungal cell and interferes with fungal RNA and protein synthesis. Of these agents, only the polyenes (amphotericin B, nystatin) depend on the amount of ergosterol incorporated into fungal cell membranes for their efficacy. These drugs bind to ergosterol molecules, forming pores in the membrane and allowing leakage of ions (especially K+) from the cells. This disruption of cell membrane integrity leads to cell lysis. An organism that decreases the amount of ergosterol in its cell membrane would likely become resistant to polyenes. Amphotericin B is a systemic antifungal active against Candida, Aspergillus, Cryptococcus, Histoplasma, Blastomyces, and Coccidioides. Nystatin is used topically; oral candidiasis is the main indication for its use ("swish and swallow").

A group of sports physicians plans to conduct a case-control study to investigate a possible association between adolescent idiopathic scoliosis (AIS) and sacroiliac joint (SIJ) dysfunction in young athletes. The case group will consist of young athletes who were diagnosed with AIS during a regular checkup by a sports physician. Which of the following is the most appropriate control group for this study?

young athletes with no diagnosis of AIS irrespective of SIJn status A case-control study is an observational design in which potential participants are initially identified as cases or controls according to the dependent variable or outcome (eg, disease of interest). Once cases and controls are identified, the presence of past exposure to ≥1 risk factors of interest is determined in each group. Finally, the frequency of exposure to the risk factors is compared between cases and controls to estimate the association between the risk factors and the outcomes. If there is a statistically significant difference in the frequency of exposure to the risk factor between the 2 groups, it is likely that the risk factor in question is associated with the disease. In this example: The population of interest is young athletes. Therefore, both the cases and control groups must consist of young athletes (Choices F and G). The disease of interest (ie, what defines a case) is adolescent idiopathic scoliosis (AIS). Therefore, the cases must have AIS and controls must not have AIS (Choices A, B, and C). The risk factor of interest is sacroiliac joint (SIJ) dysfunction. Cases and controls must be selected irrespective of SIJ status because the presence of SIJ is what is compared between cases and controls (Choice E). Therefore, the cases are young athletes with a diagnosis of AIS irrespective of SIJ status; the controls are young athletes with no diagnosis of AIS irrespective of SIJ status because what determines whether the disease (ie, AIS) is associated with the risk factor (ie, SIJ) is the difference in the frequency of the risk factor between cases and controls.

A 33-year-old woman is transferred to the university medical center from a small community hospital due to acute pyelonephritis with sepsis. She has a history of recurrent urinary tract infections due to urinary retention related to a spinal cord injury in childhood. On arrival, the patient is febrile and lethargic. She provides no additional history, and the only information available is from poorly legible handwritten notes from the originating hospital. The patient is admitted to the intensive care unit with orders to give imipenem and vancomycin. Overnight, the patient develops a rash and severe respiratory distress requiring brief mechanical ventilation. The following morning, a transcribed physician note from the originating hospital arrives by fax and indicates that the patient has an allergy to multiple antibiotics, including imipenem. When discussing events with the patient and her family, which of the following statements is associated with reduced malpractice liability for the treating clinician?

"I am sorry an error led to your allergic reaction i will do my best to help you through it."

A study is designed to evaluate the efficacy of a new drug, KM28. The study will compare KM28 plus standard care versus standard care alone with regard to decreasing the incidence of recurrent breast cancer. The Food and Drug Administration (FDA) will approve the new drug if KM28 plus standard care decreases the rate of breast cancer recurrence by at least 40% compared to standard therapy alone. The recurrence rate on standard therapy is found to be 8%. In order for the FDA to approve KM28, what is the maximal incidence of recurrent disease acceptable for women treated with KM28 plus standard therapy?

4.8%

Molecular biologists undertake a series of experiments designed to classify proteins involved in various intracellular signaling pathways. During one of the experiments, a protein mixture obtained from a cell culture is separated by gel electrophoresis and subsequently transferred to a filter membrane. Labeled double-stranded DNA probes are then used to detect a specific protein of interest in the sample. Which of the following proteins is most likely to be detected by this method? A)C-JUN B)Ras C)beta 1 receptor D)s-100

A. Southwestern blots are used to identify and isolate proteins that bind DNA. In this technique, the target protein binds to a labeled, double-stranded DNA probe that is homologous to the protein's regulatory sequence. Of the molecules listed, c-Jun is the only DNA-binding protein. c-Jun and c-Fos are nuclear transcription factors that directly bind DNA via a leucine zipper motif. The genes that code for c-Jun and c-Fos are proto-oncogenes, genes that can become oncogenes following a mutation or with constitutive expression.

A 53-year-old man comes to the physician due to frequent headaches and dizziness. He has a history of hypertension and peptic ulcer disease. His medications include daily chlorthalidone and antacids as needed. The patient's temperature is 37 C (98.6 F), blood pressure is 146/92 mm Hg, pulse is 89/min, and respirations are 16/min. His BMI is 26 kg/m2. Physical examination shows facial plethora and moderate splenomegaly. Laboratory results are as follows: Complete blood count Hemoglobin 21.5 g/dL Hematocrit 64% Erythrocytes 7.6 million/mm3 Mean corpuscular volume 90 μm3 Mean corpuscular hemoglobin 31 pg/cell Mean corpuscular hemoglobin concentration 33% Hb/cell Red blood cell distribution width 14.0% (n = 10.3%-14.1%) Platelets545,000/mm3 Leukocytes15,500/mm3 This patient most likely has a mutation in which of the following types of proteins? A) Cytoplasmic tyrosine receptor B) Intrinsic tyrosine receptor

A. The erythropoietin receptor has no intrinsic kinase activity and must interact with Janus kinase 2 (JAK2), a cytoplasmic (non-receptor) tyrosine kinase, to initiate downstream signaling. Virtually all patients with polycythemia vera have a mutation in JAK2 that causes constitutive activation of its kinase domain, resulting in clonal proliferation of myeloid cells. JAK2 mutations have also been implicated in essential thrombocythemia, primary myelofibrosis, and other myeloproliferative disorders.

A 77-year-old woman is brought to the emergency department due to fever and confusion. The patient has a history of Alzheimer dementia and lives in a nursing home. Temperature is 38.4 C (101.1 F), blood pressure is 104/74 mm Hg, pulse is 110/min, and respirations are 18/min. Suprapubic tenderness is present on physical examination. Urinalysis shows numerous bacteria and leukocytes, and urine culture grows Pseudomonas aeruginosa. Further analysis reveals that the isolated organism produces an enzyme that is located on the cytoplasmic surface of the cell membrane and that catalyzes the transfer of acetyl groups to exogenous substances. The bacteria are most likely resistant to which of the following antibiotics? A) ciprofloxacin B)ceftazdime C)pipercilin D)gentamicin

Aminoglycoside (eg, gentamicin) resistance is most commonly due to antibiotic-modifying enzymes. These enzymes add chemical groups to the antibiotic, which diminishes its ability to bind to the 16S ribosomal RNA within the 30s ribosomal subunit.

Fourth-year medical students are recruited for a research study assessing their ability to interpret biopsy samples obtained during routine clinical practice. As part of the study, they are given samples of normal respiratory mucosa and asked to identify the cell types present after staining with hematoxylin and eosin. The students observe that the respiratory epithelium changes in composition as the airways continue distally from the trachea to the alveolar ducts. Which of the following features is last to disappear?

Bronchi have a ciliated pseudostratified columnar epithelium with mucin-secreting goblet cells and submucosal mucoserous glands. The airway epithelium gradually changes to ciliated simple cuboidal by the level of the terminal bronchioles. Bronchioles lack glands and cartilage, and the number of goblet cells decreases distally, ending before the terminal bronchioles. Ciliated epithelium persists up to the respiratory bronchioles.

A 64-year-old man comes to the office due to exertional chest pain over the last 6 months. He is a lifelong 1 pack per day cigarette smoker and has a history of type 2 diabetes mellitus and peripheral artery disease. The patient undergoes treadmill exercise stress testing and develops substernal chest pain on moderate exertion accompanied by ECG changes that resolve immediately upon rest. He refuses invasive cardiac testing. The patient is started on low-dose aspirin therapy for secondary prevention of cardiovascular disease but experiences shortness of breath and wheezing with the medication. Which of the following is the best alternate therapy for this patient?

Clopidogrel Clopidogrel irreversibly blocks the P2Y12 component of ADP receptors on the platelet surface and prevents platelet aggregation. Clopidogrel is as effective as aspirin in the prevention of cardiovascular events in patients with coronary heart disease.

11-year-old boy is brought to the emergency department by ambulance after he briefly lost consciousness. The patient's mother found him on the kitchen floor shaking and jerking. She believes that he was out of her sight for only a few minutes. When asked about any other abnormal behaviors, she says that her son often stares into space and does not respond to questions during these episodes.Several hours later, he is alert and says that he wants to go home. What is the best long term treatment? What is the pathogenesis and the treatment (MOA) for this?

Juvenile absence epilepsy + Tonic clonic seizure. Absence seizures: momentary lapses in consciousness (~10 seconds) associated with a classic 3-Hz spike-wave pattern on EEG. Younger children with absence seizures typically do not develop other seizure types; however, later onset (eg, age 10-12) of absence epilepsy is associated with an increased incidence of generalized onset tonic-clonic or myoclonic seizures. He requires a broad spectrum medication: Valproate, levetiracetam, topiramate and lamotrigine. Valproate MOA: Increases Na channel inactivation, Increases GABA levels in CNS. Uses: preferred in grand mal epilepsy over phenytoin, and in petit mal over ethosuxomide. Also preferred in myoclonus, atonic and infantile spasm. Alternative to Lithium in bipolar disorder and migraine prophylaxis. AE: GI distress, increased appetite and weight gain, Tremors, fatal hepatotoxicity, Pancreatitis, Teratogenic (Neural tube defects), CYP inhibitor

A 40-year-old man comes to the office due to erectile dysfunction and loss of libido. The patient's history is notable for Hodgkin lymphoma that was treated with chemotherapy. Physical examination is normal except for small testes. Laboratory evaluation shows consistently low total and free early morning testosterone levels and elevated serum LH. Transdermal testosterone therapy is prescribed. Which of the following should be periodically monitored during this patient's treatment?

Testosterone replacement therapy is indicated for men with symptomatic hypogonadism. It is associated with an increase in serum prostate-specific antigen and may raise the risk of prostate cancer. In addition, testosterone therapy can cause erythrocytosis (leading to thromboembolism), and hematocrit should be regularly monitored.Also avoid in pt with severe obstructive sleep apnea

A group of psychiatrists conducts a double-blind placebo-controlled randomized trial to determine the efficacy and safety of cariprazine for the treatment of depressive episodes associated with bipolar I disorder (bipolar depression) in adults. The treatment response rate was 46% for cariprazine versus 32% for placebo, and the rate for all adverse events was 7% for cariprazine versus 5% for placebo. The differences in rates were statistically significant. Which of the following is the approximate number of patients who must be exposed to cariprazine to cause harm to 1 person who otherwise would not have been harmed?

The question is asking for number needed to harm NNH: Number of patients who need to be exposed to a risk factor for 1 patient to be harmed. Higher number=safe exposure NNH:1/Attributable risk or AR AR: Is the difference in risk between exposed and unexposed groups. AR: a/a+b - c/c+d AR%: RR-1/RR AR: 7-5=2 NNH: 1/2=0.5=50% approximately 50 patients would need to be treated with cariprazine to cause harm to 1 person who otherwise would not have been harmed. The lower the NNH, the more risk of harm; an NNH of 1 means that every patient treated is harmed.

A 4-day-old infant is brought to the emergency department with abnormal movements. The patient has had intermittent episodes of tonic posturing over the past 3 hours as well as poor feeding, vomiting, and irritability for the past 2 days. The mother also reports that his diapers smell like "caramelizing sugar." There are no known medical problems in the family, but the child's maternal aunt died "sometime in the first year" of life from unknown causes. Examination shows a lethargic infant with intermittent posturing episodes and increased generalized muscle tone. Laboratory studies of plasma and urine confirm the diagnosis. In addition to appropriate dietary restriction, supplementation with which of the following may improve this infant's condition?

This infant has symptoms typical of maple syrup urine disease (MSUD), an autosomal recessive disorder characterized by the defective breakdown of branched-chain amino acids (leucine, isoleucine, and valine). Degradation of these 3 amino acids first involves transamination to their respective α-ketoacids, which are subsequently metabolized by several enzymes referred to as the branched-chain α-ketoacid dehydrogenase complex (BCKDC). Mutations in BCKDC result in accumulation of the branched-chain amino acids in serum and peripheral tissues, resulting in neurotoxicity that includes seizures, irritability, lethargy, and poor feeding. A metabolite of isoleucine gives the urine a distinctive sweet odor. MSUD can be life-threatening if untreated, as brain swelling may lead to death. Branched-chain α-ketoacid dehydrogenase (in addition to pyruvate dehydrogenase and α-ketoglutarate dehydrogenase) requires 5 cofactors: Thiamine, Lipoate, Coenzyme A, FAD, NAD (mnemonic: Tender Loving Care For Nancy). Some patients with MSUD improve with high-dose thiamine treatment, but most still require lifelong dietary restrictions.

A 32-year-old man comes to the hospital due to 1 week of progressive shortness of breath and cough. He also reports sharp right-sided chest pain that is worsened by deep inspiration. Examination reveals decreased tactile fremitus, dullness on percussion, and decreased breath sounds over the right lower lung. Chest x-ray shows consolidation of the right lower lobe and a right-sided effusion. A thoracentesis is performed, during which the needle is inserted along the upper border of the 10th rib at the right midaxillary line. Which of the following structures is most at risk of being injured as a result of the intervention?

Thoracentesis should be performed below the 6th rib in the midclavicular line, the 8th rib along the midaxillary line, or the 10th rib along the paravertebral line in order to minimize the risk of lung injury. Insertion of a needle lower than 9th rib increases the risk of penetrating abdominal structures. The needle should also be inserted along the upper border of the rib to prevent injury to the intercostal vessels. Remember that intercostal VAN lie in the subcostal groove along the lower border of the lung

A group of investigators is studying the mechanisms of signal transduction involved in the pathogenesis of type 2 diabetes mellitus. Their research focuses on the intracellular signaling cascade that begins when insulin binds to the insulin receptor. In a series of experiments, they demonstrate that pre-treatment with tumor necrosis factor alpha (TNF-α) results in decreased insulin-mediated glucose uptake. This effect of TNF-α is most likely mediated through upregulation of which of the following processes? A) Cyclic AMP hydrolyzation B)Phosphotidal inositol hydrolysis C) Serine residues phosphorylation D) proline residue hydroxylation

Tumor necrosis factor alpha (TNF-α) is a proinflammatory cytokine that induces insulin resistance through the activation of serine kinases, which then phosphorylate serine residues on the beta subunits of IR and IR substrate-1. This inhibits tyrosine phosphorylation of IR substrate -1 by IR and subsequently hinders downstream signaling, resulting in resistance to the normal actions of insulin. Phosphorylation of threonine residues has similar effects. Catecholamines, glucocorticoids, and glucagon can also induce insulin resistance by this same mechanism.

A 50-year-old man is brought to the emergency department due to a severe, sudden-onset headache that started an hour ago. The patient reports that he has had mild headaches and decreased libido over the past 3 months. He has no other medical conditions and takes no medications. Physical examination reveals bilateral deficits involving the temporal visual fields and impaired extraocular eye movements. Shortly after being admitted to the hospital, he becomes acutely hypotensive and loses consciousness. The patient dies despite aggressive resuscitation efforts. Which of the following is most likely to be found on autopsy?

acute infarction of the pituitary Acute pituitary hemorrhage (pituitary apoplexy) is characterized by severe headaches, bitemporal hemianopsia (compression of the optic chiasm), and ophthalmoplegia (compression of the oculomotor nerve). It usually occurs in a preexisting pituitary adenoma. Pituitary apoplexy is a medical emergency that requires urgent treatment with glucocorticoids to prevent acute adrenal crisis and circulatory collapse.

A 43-year-old woman comes to the emergency department due to low-grade fevers and malaise. She has a history of lung transplantation due to cystic fibrosis and had 2 episodes of cytomegalovirus (CMV) viremia over the past few months, which were treated with ganciclovir. Temperature is 37.6 C (99.7 F). Physical examination is unrevealing. Chest x-ray and urinalysis are negative for infection. Polymerase chain reaction testing for CMV in blood shows elevated CMV levels consistent with CMV viremia. Given the recurrent episodes of viremia, genotype analysis is performed and demonstrates ganciclovir-resistant CMV. The patient is started on an alternate intravenous antiviral agent. She develops hypocalcemia and hypomagnesemia. Which of the following agents is the most likely cause of these side effects?

foscarnet Foscarnet is a pyrophosphate analog that is sometimes used for ganciclovir-resistant cytomegalovirus (CMV) infections. Foscarnet can chelate calcium. Moreover, foscarnet-induced renal wasting of magnesium may lead to hypomagnesemia and a reduction in the release of parathyroid hormone, which contributes to the hypocalcemic state. Both hypocalcemia and hypomagnesemia can promote seizures.

A 43-year-old previously healthy man is hospitalized after sustaining a head injury in a motor vehicle collision. Several days later, the patient develops worsening serum electrolyte disturbances. Further evaluation reveals the cause is inappropriate antidiuretic hormone secretion. Treatment with a vasopressin V2 receptor antagonist is initiated. Which of the following changes are most likely to occur as a direct result of the administered medication? Plasma osmolality, Urine Na excretion and urine output

increase, no change, increase

Healthy adult volunteers are enrolled in a phase I clinical trial investigating the properties of a newly developed oral antimicrobial agent. The drug is administered in different amounts to the volunteers over the course of several weeks to determine the best dosage that minimizes toxicity while maintaining trough levels above the minimum inhibitory concentration. While reviewing the data, the researchers note that the drug's half-life seems to vary amongst the study participants. An increase in which of the following pharmacologic parameters is most likely responsible for the longer half-life seen in certain individuals?

volume of distribution b/c t1/2=0.7xvd/cl

A 68-year-old woman comes to the office due to a month-long history of worsening fatigue, shortness of breath, and dry cough. The patient has symptoms that worsen with exertion and when lying flat. She has had no chest pain, palpitations, or lightheadedness. The patient has a history of hypertension and breast cancer that is in remission after surgery and doxorubicin-based chemotherapy 15 years ago. She does not use tobacco, alcohol, or illicit drugs. Blood pressure is 110/62 mm Hg and pulse is 94/min. Physical examination shows crackles at the lung bases and bilateral pedal edema. Which of the following sets of findings are most likely present in this patient? WHAT WOULD HAPPEN TO HER RAP AND EDLVP

both will be increased The fact she has pulmonary edema means her left vent pressure is high and the fact that she has pedal edema means that her right atrium p is high Patients with decompensated heart failure have elevated left ventricular end-diastolic pressure and decreased cardiac output that is most often primarily due to left ventricular dysfunction. Right atrial pressure (ie, central venous pressure) is also elevated in advanced heart failure due to volume overload; right-sided heart failure (most often occuring secondary to left-sided failure) can also contribute to elevated right atrial pressure.

A 63-year-old man comes to the office for a follow-up examination of hypertension. At his last appointment, he revealed that he had increased his intake of 12-oz cans of beer from 3 to 6 cans a day due to stress at work. The physician advised him to seek help for his alcohol use as it could be causing his elevated blood pressure and has many negative health risks. The patient now tells the physician, "I thought about what you said. I know my alcohol use has gotten out of hand and is affecting my health. My wife and daughter also say that I need to quit. I have decided to look into options for treatment in my area." Which of the following best describes this patient's stage of behavioral change? A) contemplation B) action C) preparation D) precontemplation E) maintanance

c Making changes is difficult and takes time and dedication. Most people move through 5 distinct stages of change, beginning with initial denial of the problem (precontemplation), moving to acceptance that there is a problem (contemplation), and then planning (preparation) and putting into action changes that deal with the problem. Finally, these changes require maintenance over the long term. Patients often relapse and may have to repeat a certain stage(s) of change a number of times. This is part of the process and should be anticipated. This patient has acknowledged that he has a drinking problem, has thought about making a change, and is preparing to take steps to manage the problem by seeking out available resources. In the preparation stage, the physician should encourage and facilitate the patient's plan to change (eg, reinforce his commitment to action, assist in identifying treatment resources).

A 48-year-old man comes to the office due to several hours of severe right knee pain. The patient has a history of peptic ulcer disease and gastroesophageal reflux disease. His right knee is swollen, erythematous, and tender. Arthrocentesis is performed and synovial fluid analysis shows needle-shaped, negatively birefringent crystals with many neutrophils. The medication given to this patient selectively binds to an interleukin-1 inducible enzyme that is highly expressed by inflammatory cells and undetectable in the surrounding normal tissue. Which of the following is most likely the drug used in this patient's treatment?

celecoxib Cyclooxygenase (COX), also known as prostaglandin endoperoxide synthase, catalyzes the conversion of arachidonic acid into prostanoids. It exists in 2 isoforms designated COX-1 and COX-2. COX-1 is constitutively expressed in various tissues and is involved in a number of "housekeeping functions" (eg, platelet aggregation, gastric mucosal protection, vascular homeostasis). COX-2 is an inducible enzyme that is undetectable in most tissues under normal conditions. During inflammation, infiltrating cells secrete cytokines (eg, interleukin-1, TNF-α) that cause COX-2 upregulation in the inflamed tissue. COX-2 then drives the synthesis of pro-inflammatory arachidonic acid metabolites. This patient's acute gout attack should be treated with an agent that provides prompt pain relief without exacerbating his gastrointestinal problems. Nonsteroidal anti-inflammatory drugs (NSAIDs) such as ibuprofen, indomethacin, and naproxen inhibit both COX-1 and COX-2 and can cause significant gastrointestinal injury. Selective COX-2 inhibitors (eg, celecoxib) decrease inflammation but have no effect on COX-1, minimizing gastroduodenal toxicity.

A 25-year-old Caucasian man is undergoing evaluation for azoospermia. The patient has been monogamous with his long-term girlfriend and does not use contraception during sexual intercourse. They have been trying to conceive for the past year with no success. The patient has a past medical history of recurrent pneumonia with frequent hospitalizations for antibiotic treatment. He takes no medications and does not use tobacco, alcohol, or illicit drugs. The patient has no allergies and his immunizations are up-to-date. His family history is unknown as he was adopted as an infant. Physical examination shows digital clubbing. A transrectal ultrasound shows bilateral absence of the vas deferens. Which of the following tests would most likely confirm the underlying diagnosis of this patient's condition? Dx?

chloride sweat test Cystic fibrosis CFTR gene mutations are the most common cause of congenital bilateral absence of the vas deferens (CBAVD). Patients with CBAVD have azoospermia and infertility but normal levels of FSH, LH, and testosterone. Elevated sweat chloride levels are diagnostic of cystic fibrosis.

In Duchenne muscular dystrophy, when the patient becomes symptomatic, muscles near which of the following joints will be most likely effected first? A) hips B)ankles C) wrist D) Shoulders

Pelivic/hips DMD is an x linked recessive disorder due to absence of dystropin gene, recall patients have +gower sign where they use their upper limbs to get off from seated position the reason they do this is because their hip muscles are very weak.

A 35-year-old woman comes to the emergency department with nausea, vomiting, and fever. Her symptoms began 24 hours ago, and she has been unable to eat or drink anything since. She has a 3-year-old daughter who had similar symptoms 2 days earlier but is now fine. Laboratory studies show a blood glucose level of 82 mg/dL despite her 24-hour fast. Maintenance of this patient's blood glucose levels is facilitated by hepatic conversion of pyruvate into glucose. Which of the following substances directly stimulates the first enzyme involved in this process? A) Lactate B) oxaloacetate C) pyruvate D) Acetyl coA e) Fructose 2,6 bisphosphate

-D During gluconeogenesis, substances such as lactate and alanine are converted to pyruvate. However, pyruvate cannot be converted to phosphoenolpyruvate directly as pyruvate kinase is unidirectional. To convert pyruvate to phosphoenolpyruvate, pyruvate first undergoes biotin-dependent carboxylation to oxaloacetate in the mitochondria. This reaction is catalyzed by pyruvate carboxylase. The activity of pyruvate carboxylase is increased by acetyl-CoA. This critical regulatory step diverts pyruvate to pyruvate dehydrogenase when acetyl-CoA levels are too low, preventing the cell from becoming energy starved. When acetyl-CoA levels are high (as with increased beta oxidation of fatty acids during fasting), pyruvate carboxylase can operate at full capacity and convert most of the pyruvate into oxaloacetate for use in gluconeogenesis.

A study was conducted to assess the age at menarche among young female gymnasts. Based on commitment to the sport, gymnasts were divided into two groups Competitive: sample size 16 mean= 13.4 SD= 1.3 Assuming that age at menarche is normally distributed, which of the following is closest to the probability that a randomly chosen competitive gymnast will have onset of menarche at age ≥16?

0.025

A 16-year-old boy with suicidal thoughts is brought to the office by his parents. He has been in constant conflict with his parents since refusing to stop drinking alcohol, which he does on a daily basis. The patient is the oldest of 6 children; his mother suffers from lower back pain and relies on him for child care, which he finds overwhelming. His father works 2 jobs and is often not around to help with the children. His mother keeps a firearm by her bedside for protection when she is alone with the children at night. The patient dropped out of high school last year after getting dumped by his girlfriend. He recently contracted Lyme disease with resultant symptoms of fatigue and myalgia. Which of the following interventions is the best next step to decrease this patient's risk of completed suicide? A) Advise mother to lock away her firearms B) Start txt for depression C) Start alcohol abuse counseling D) Support pt return to highschool

A Access to firearms greatly increases the risk of completed suicide. Evaluation of a patient's access to guns is a key part of suicide risk assessment. Other interventions to reduce suicide risk include decreasing stress, increasing psychosocial support, treating psychiatric illness and substance use, and managing pain.

A 3-week-old boy with discharge from the umbilicus is brought to the clinic by his parents. His postnatal course was uncomplicated, with shriveling of the cord around 14 days of life. Vital signs are normal. Examination of the area reveals a small reducible umbilical hernia, minimal clear to straw-colored discharge from the umbilicus, and erythema around the area. Laboratory results are as follows: Hemoglobin 12 g/dL Hematocrit 36% Leukocytes 11,000 cells/mm3 Neutrophils 50% Lymphocytes 45% Which of the following is the most likely cause of this child's condition? A) absence of neutrophil migration B) incomplete closure of abdominal wall C) persistence of allontois remenant D) persistent of omphalomesenteric duct

Around 3 weeks gestation, the yolk sac forms a protrusion (allantois) that extends into the urogenital sinus. The upper part of the urogenital sinus gives rise to the bladder. The allantois, which originally connected the urogenital sinus with the yolk sac, becomes the urachus, a duct between the bladder and the yolk sac. Failure of the urachus to obliterate before birth leads to several abnormalities: Complete failure of obliteration of the urachus results in a patent urachus that connects the umbilicus and bladder. Patients present with straw-colored urine discharge from the umbilicus, which is exacerbated by crying, straining, or prone position. Local skin irritation can cause erythema. Failure to close the distal part of the urachus (adjacent to the umbilicus) results in a urachal sinus. This presents with periumbilical tenderness and purulent umbilical discharge due to persistent and recurrent infection. Failure of the central portion of the urachus to obliterate leads to a urachal cyst.

A large cohort study is conducted to assess the association between smoking and squamous cell carcinoma of the esophagus among middle-aged Chinese men. During 10 years of follow-up, smokers have 5 times the risk of esophageal carcinoma compared to non-smokers (relative risk = 5.0, 95% confidence interval = 2.9-7.1). According to the study results, what percentage of squamous cell carcinoma of the esophagus in smokers can be attributed to smoking?

Attributable risk: Is the difference in risk between exposed and unexposed groups. AR: a/a+b - c/c+d AR%: RR-1/RR The attributable risk percent in the exposed is an important measure of the impact of a risk factor. ARPexposed = 100 × [(risk in exposed - risk in unexposed)/risk in exposed] This basic definition can be used to derive an equivalent formula involving relative risk (RR): ARPexposed = 100 × [(RR - 1)/RR], where RR = risk in exposed/risk in unexposed Applying the formula to this example: ARPexposed = 100 × [(RR - 1)/RR] = 100 × [(5 - 1)/5] = 100 × (4/5) = 100 × 0.8 = 80%

A researcher is studying the role of glucose metabolites in normal cellular function. A specific human cell type is incubated in glucose-rich media. Intracellular levels of glucose metabolizing enzymes, intermediate products, and generated ATP are measured. In these cells, glycolysis of a single glucose molecule always yields pyruvate but sometimes generates no net ATP. Which of the following cells is most likely being studied in this experiment? A)hepatocytes B) erythrocytes C) neurons D) adipocytes

B 2,3-BPG decreases hemoglobin's affinity for oxygen. Therefore, in the presence of lower blood oxygen concentrations, higher 2,3-BPG levels within erythrocytes enable increased oxygen delivery in the peripheral tissues. 2,3-BPG is produced from 1,3-BPG by the enzyme bisphosphoglycerate mutase. This reaction bypasses an ATP-generating step of glycolysis, causing no net gain in ATP.

A 31-year-old man is hospitalized after sustaining multiple injuries during a motor vehicle collision. The patient is not conscious initially, and no information is available regarding his medical history. Two days after initial hospitalization, he experiences palpitations and difficulty sleeping. Shortly thereafter, the patient suffers a generalized tonic-clonic seizure. When he recovers, he tells the physician that he usually takes medications for anxiety, bipolar disorder, and chronic low back pain. Temperature is 36.7 C (98.1 F), blood pressure is 129/82 mm Hg, pulse is 92/min, and respirations are 14/min. He appears anxious, tremulous, and diaphoretic. Withdrawal from which of the following medications is the most likely explanation for this patient's presentation?

Benzodiazepine withdrawal is characterized by anxiety, tremor, insomnia, and sympathetic hyperactivity (eg, diaphoresis, palpitations). Severe benzodiazepine withdrawal may also be accompanied by psychosis, seizures, or death. Mostly seen with short acting benzos ending in -olam but lorazapem was the answer and it was the only benzo in the answer choice.

A 45-year-old woman comes to the clinic with a 3-month history of persistent shoulder and back pain and easy fatigability. She also has pain and stiffness in all her muscles that worsens in the morning and evening. She does not participate in any regular exercise because her pain is exacerbated with activity. She works as a computer programmer and reports some difficulty concentrating on her job. Past medical history is significant for depression and gastroesophageal reflux disease, but she is not currently taking any medications. Physical examination shows normal range of motion and 5/5 muscle strength in both upper and lower extremities. Soft tissue tenderness is present at several locations bilaterally above and below the waist. Which of the following is the most likely diagnosis? A) dermatomyositis B)polymyalgia rheumatica C) fibromyalgia

C Clinical features of fibromyalgia Symptoms: Widespread musculoskeletal pain, Fatigue, Impaired attention & concentration Psychiatric disturbances (eg, depression, anxiety) Symptoms lasting for ≥3 months Physical examination: Multiple tender points at characteristic locations, absence of joint or muscle inflammation Laboratory findings: Normal acute phase reactants (eg, ESR, CRP) & other inflammatory markers Treatment: Progressive exercise regimen Medications: tricyclic antidepressants, cyclic skeletal muscle relaxants, serotonin-norepinephrine reuptake inhibitors

A 51-year-old man comes to the emergency department with sudden-onset, sharp, right flank pain; nausea; and vomiting. He has had no dysuria or hematuria. The patient has no past medical conditions and takes no daily medications. He does not use tobacco, alcohol, or illicit drugs. Temperature is normal. On examination, the patient appears to be in severe pain and cannot find a comfortable position on the bed. There is no abdominal rigidity or rebound, and no masses are palpable. Cardiopulmonary examination is normal. There is no peripheral edema. Imaging shows a 1-cm calculus in the right proximal ureter at the level of the L3 vertebra; the renal pelvis and proximal ureter are dilated, as shown in the exhibit. Which of the following is most likely increased in this patient's right kidney? A) Bowmans space hydrostatic pressure B) Glomerulus filteration C) Tubular hydrostatic pressure

C The glomerular filtration rate depends on the permeability of the capillary wall and the net ultrafiltration pressure. Net ultrafiltration pressure is a result of pressure gradients formed by Starling forces: The hydrostatic pressure gradient is the difference between the hydrostatic pressure in the intraglomerular capillaries and the Bowman space. Typically, the hydrostatic pressure in the glomerular capillaries is markedly greater than the pressure in the Bowman space, favoring filtration. The oncotic pressure gradient is the difference between the oncotic pressure in the intraglomerular capillaries and the Bowman space. Oncotic pressure is chiefly driven by large plasma proteins (eg, albumin), which do not freely filter across the glomerular capillary basement membrane due to both a size and charge barrier. Therefore, the oncotic pressure is negligible within the Bowman space, favoring absorption of fluid into the glomerular capillaries. This patient has a kidney stone in the right ureter; the hydroureter and hydronephrosis suggest acute urinary tract obstruction. The resultant reflux of urine backward into the relatively noncompliant renal tubules results in increased renal tubular hydrostatic pressure. As the intraglomerular capillary hydrostatic pressure is unchanged, this results in a decreased hydrostatic pressure gradient leading to a reduction in glomerular filtration

A 68-year-old man comes to the office due to several weeks of progressive exertional dyspnea and lower extremity edema. Medical history is significant for non-Hodgkin lymphoma, which is in remission after chemotherapy 8 years ago. Blood pressure is 126/76 mm Hg and pulse is 88/min. Auscultation reveals bibasilar lung crackles and 1+ bilateral lower extremity edema. Echocardiogram shows biventricular dilation and a left ventricular ejection fraction of 35%. Stress myocardial perfusion scan is negative for inducible ischemia. After initial stabilization, long-term use of which of the following medications will most likely improve survival in this patient?

Carvidolol Drugs that have been shown to improve long-term survival in patients with heart failure due to left ventricular systolic dysfunction include beta blockers, ACE inhibitors, angiotensin II receptor blockers, and aldosterone antagonists.

A 24-year-old woman with a medical history of bronchial asthma comes to the office due to shortness of breath and wheezing for the past 2-3 days. She developed a cold 4 days ago and is not feeling well. The patient has a nebulizer at home and used multiple doses of albuterol with little response prior to arriving. She takes no other medications. The patient is diaphoretic and in moderate respiratory distress. Respiratory examination shows bilateral wheezing, diffusely decreased breath sounds, and increased use of accessory muscles of respiration. Laboratory results reveal a serum potassium of 3 mEq/L. Which of the following mechanisms is the most likely cause of this patient's hypokalemia?

Causes of hypokalemia Decreased intake: Starvation, anorexia Intracellular translocation Insulin (eg, treatment of DKA, refeeding syndrome) β-adrenergic activityPharmacologic (eg, albuterol, dobutamine)Stress-induced (eg, alcohol withdrawal, acute MI) Alkalosis (respiratory or metabolic) ↑ Cell reproduction (eg, acute myeloid leukemia, GM-CSF) Gastrointestinal loss: Diarrhea, vomiting, hyperaldosteronism Urinary loss: Hyperaldosteronism, diuretics, RTA types 1 and 2 Sweat loss: Extreme exercise in hot climate DKA = diabetic ketoacidosis; MI = myocardial infarction; GM-CSF = granulocyte-macrophage colony-stimulating factor; RTA = renal tubular acidosis. Potassium is primarily stored intracellularly (~98% of total body stores) through the action of the Na-K-ATPase pump. Beta-adrenergic activity increases the activity of the Na-K-ATPase pump; therefore, both endogenous catecholamines and therapeutic beta-2 agonists (eg, albuterol, dobutamine) can cause transient hypokalemia due to increased transport of potassium intracellularly

A 54-year-old woman comes to the physician because of multiple firm violaceous nodules on her right upper arm. She was diagnosed with right-sided breast carcinoma twelve years ago and was treated with radical mastectomy. Extensive axillary lymph node dissection performed at that time revealed no lymph node metastases. Post-operatively, the patient developed chronic lymphedema involving her right arm. Which of the following is the most likely etiology of her current skin lesions?

Chronic lymphedema is a risk factor for the development of cutaneous angiosarcoma also known as Stewart-Treves syndrome. Radical mastectomy with axillary lymph node dissection is a classic predisposing procedure, although any form of chronic lymphedema can be implicated. Histopathologically, angiosarcoma will show infiltration of the dermis with slit-like abnormal vascular spaces. The prognosis for patients with angiosarcoma is poor because the tumor is usually widespread by the time of diagnosis. There is controversy regarding use of the term angiosarcoma versus lymphangiosarcoma, as it appears that this entity arises from blood vessels rather than from lymphatic vessels.

A pediatric cardiologist decides to determine whether there is a relationship between body composition and blood pressure in a random sample of children and adolescents age 5-15. He used standard procedures to collect anthropometric measurements that included weight, height, hip circumference (HC), and waist circumference (WC). Systolic blood pressure (SBP) and diastolic pressure (DBP) readings were taken at least 3 times at 5-minute intervals after the participants had been seated, and average SBP and DBP readings were calculated based on these measurements. Which of the following statistical tests is adequate to determine whether there is a relationship between WC and average DBP? A) Variance analysis B) t test C) chi square D)meta analysis E) Correlation analysis

Correlation analysis is a statistical technique used to assess the strength and direction of a linear relationship between 2 quantitative variables (often, but not always, a dependent and an independent variable). In this study: The quantitative dependent variable was diastolic pressure (DBP). The quantitative independent variable was waist circumference (WC). Correlation analysis can determine whether there is a relationship between DBP and WC

A 36-year-old man comes to the office after he was found to have an abnormal lipid panel during employee wellness testing at his company. He has no prior medical problems and takes no medications. The patient is a software technician and has a sedentary lifestyle. He eats mostly fast foods, rarely exercises, and drinks 2-3 cans of beer daily. His BMI is 31 kg/m2. Physical examination is unremarkable. Results of laboratory studies performed in the office are as follows: Total cholesterol290 mg/dL High-density lipoprotein45 mg/dL Low-density lipoprotein110 mg/dL Triglycerides675 mg/dL Lifestyle modification with a balanced diet, regular exercise, and reduced alcohol intake is advised. He is also started on fenofibrate therapy. This medication is most likely to help the patient by which of the following mechanisms?

Fibrates lower triglyceride levels by activating peroxisome proliferator-activated receptor alpha, which leads to decreased hepatic VLDL production and increased lipoprotein lipase activity. Fish oil supplements containing high concentrations of omega-3 fatty acids lower triglycerides by decreasing production of VLDL and apolipoprotein B.

Researchers develop a novel drug to treat HIV infection. In an in vitro experiment, wild-type viral isolates are cultured with human CD4 T lymphocytes in the presence of the drug. Microscopic evaluation of these cells reveals no cytopathic changes, and no intracellular viral particles can be detected. Further analysis reveals that in the presence of the drug the virus attaches to the cellular membrane, but fusion of the viral and cell membranes does not occur. Which of the following is the most likely target of this medication? A) Viral nef gene product B) viral env gene product C) T cell antigen- receptor CD3 complex D) Tcell chemokine receptor

HIV attaches to host cells using the viral surface glycoprotein gp120. This glycoprotein binds to the CD4 molecule as the primary receptor and the chemokine receptor CCR5 (or CXCR4) as a coreceptor. Binding of the primary receptor and coreceptor induces a conformational change in gp120 that exposes the underlying transmembrane glycoprotein gp41, which mediates viral fusion to the host cell and release of the viral capsid into the cytoplasm. The drug described in this case does not interfere with viral attachment but does interfere with viral fusion. This implies that gp120 is still able to bind to the primary CD4 receptor but may be blocked from binding to the chemokine coreceptor, which would prevent the conformational change to gp120 required for viral fusion. Chemokine receptor antagonists (eg, maraviroc) block this step in the HIV replication cycle. The drug described in this case could also be a fusion inhibitor (eg, enfuvirtide), which blocks gp41 from fusing the viral and host plasma membranes.

A 51-year-old woman comes to the office for a follow-up visit. Three weeks ago, she had a screening mammogram that was suspicious for malignancy. An ultrasound-guided needle aspiration was performed and showed invasive ductal carcinoma. The patient underwent a lumpectomy with axillary dissection and was found to have no metastatic disease. Immunohistochemical analysis of the tumor specimen was significant for the following: Estrogen receptorPositiveProgesterone receptorPositiveHuman epidermal growth factor receptor 2Positive Adjuvant therapy with a monoclonal antibody is started. Which of the following is the most likely target of this drug? A) estrogen receptor B) aromatase inhibitor C) tyrosine receptor

Human epidermal growth factor receptor 2 (HER2) is a tyrosine kinase receptor that is overexpressed in 20% of breast cancer patients. In such patients, HER2 blockade with the monoclonal antibody trastuzumab downregulates cellular proliferation and promotes apoptosis.

A 67-year-old man comes to the office due to increased fatigue and weight gain over the past 6 months. He has hypertension controlled with a calcium channel blocker. Two years ago, the patient's physical examination was unremarkable, and his blood tests, including a lipid panel, were normal. Blood pressure is 135/98 mm Hg, and pulse is 62/min. BMI is 28 kg/m2. On physical examination, the thyroid appears to be mildly enlarged and is irregular and firm in consistency. Laboratory studies reveal elevated TSH, low free thyroxine (T4), high total and LDL cholesterol, and mildly elevated triglyceride levels. Which of the following is the most likely underlying mechanism for the elevated cholesterol in this patient? A) Activation of PPAR ALPHA B) decreased LDL gene receptor gene expression C) induction of HMG coA reductase D)upregulation of LDL receptor degradation

Hypothyroidism causes decreased expression of LDL receptors in the liver, leading to decreased clearance of LDL and increased blood LDL levels. It also causes hypertriglyceridemia due to decreased expression of lipoprotein lipase. The adverse effects of hypothyroidism on lipid levels can increase the risk of coronary atherosclerosis.

A 52-year-old man comes to the office after his wife noted that his eyes appeared different. The patient reports no symptoms. Medical history is significant for hypertension and type 2 diabetes mellitus. He has smoked 1.5 packs of cigarettes daily for 25 years. Blood pressure is 140/86 mm Hg and pulse is 84/min. On physical examination, the right pupil is 3 mm and the left pupil is 5 mm. When the light in the examination room is dimmed, the anisocoria increases. Which of the following is the most likely location of the lesion?

In dim light, your pupils dilate to let more light in this is mediated by SNS. In bright light, your pupils will constrict this is mediated by PNS. In this patient there is an inability of the right pupil to dilate upon darkness aka dim light. So the lesion is in the Right oculosympathetic pathway Determining which pathway is affected in anisocoria can be accomplished by observing the change in pupillary discrepancy in both bright and dim light: Asymmetry that increases in dim light indicates that the smaller pupil is unable to dilate (ie, miosis) due to loss of sympathetic nerve input. Under bright light, the asymmetry will decrease because parasympathetic input (pupillary constriction) is unaffected. Asymmetry that increases under bright light indicates that the larger pupil is unable to constrict (ie, mydriasis) due to loss of parasympathetic nerve input. In dim light, the asymmetry will decrease because sympathetic input (pupillary dilation) is unaffected.

A 6-year-old girl with chronic anemia requiring repeated blood transfusions is undergoing genetic testing. The patient's mother and older sibling have a history of mild anemia. Her peripheral blood smear shows hypochromic, microcytic red blood cells, and hemoglobin electrophoresis reveals a predominance of hemoglobins F and A2. Sequencing of the β-globin gene is performed using the patient's erythroblast DNA. A schematic representation of the gene and its transcribed RNA is shown in the image below. CAT BOX gene The base sequence indicated by the bold red arrow is responsible for which of the following functions?

Initiation of gene transcription. The TATA and CAAT boxes are promoters of transcription in eukaryotic cells and are located approximately 25 and 75 bases upstream from the transcription start site, respectively. They promote initiation of transcription by serving as binding sites for transcription factors and RNA polymerase II.

A 54-year-old man comes to the emergency department due to severe fatigue and dyspnea. He has a long history of progressively worsening heart failure that has been resistant to treatment with medications, including diuretics. He was treated with chest radiation 10 years ago for non-Hodgkin lymphoma and has been in remission since then. The patient is admitted to the hospital, but his condition continues to deteriorate despite aggressive therapy. He dies 3 days later, and an autopsy is performed. Gross inspection of the heart shows dense, thick, fibrous tissue in the pericardial space between the visceral and parietal pericardium. Which of the following signs would most likely have been detected during a physical examination of this patient just prior to his death?

Kaussmal sign The autopsy finding of thick, fibrous tissue in the pericardial space is consistent with constrictive pericarditis, a potential complication of chest radiation therapy for non-Hodgkin lymphoma. This dense, rigid pericardial tissue encases the heart and restricts ventricular filling, causing low cardiac output (manifesting with fatigue and dyspnea on exertion) and progressive right-sided heart failure (manifesting with hepatomegaly and peripheral edema). Physical examination in constrictive pericarditis typically shows elevated jugular venous pressure (JVP) with prominent x and y descents and a pericardial knock (early diastolic sound that occurs before S3) and may also demonstrate pulsus paradoxus (>10 mm Hg drop in systolic blood pressure during inspiration). In addition, Kussmaul sign may be present. Under normal circumstances, the decrease in intrathoracic pressure during inspiration increases venous return to the right side of the heart and lowers JVP. However, in constrictive pericarditis, the rigid pericardium prevents the right side of the heart from accommodating increased venous return, which leads to a paradoxical rise in JVP during inspiration, referred to as Kussmaul sign.

A 67-year-old man comes to the office after he noticed several nodules in his left axilla. He has a history of malignant melanoma on his upper back, which was treated with wide surgical excision 4 years ago. The patient undergoes biopsy of an axillary lesion, and histopathology reveals melanoma recurrence. Subsequent whole-body positron emission scan shows diffuse metastatic disease involving the lungs, liver, and left axillary nodes. Checkpoint inhibitor therapy with pembrolizumab is initiated, which leads to significant reduction in the size of the axillary nodules and metastatic lesions. Which of the following best correlates with the treatment response in this patient?

Neoplastic cells generally accumulate genetic mutations that allow them to thwart the cytotoxic T-cell response and evade detection/destruction. One way to do this is by over expression of programmed death-ligand 1 (PD-L1) on the cancer cell surface; this ligand binds to the PD-1 receptor on cytotoxic T cells and suppresses their ability to induce apoptosis (T-cell exhaustion). Monoclonal antibodies that block PD-1 such as pembrolizumab have shown promise in cancers that overexpress PD-L1 (eg, melanoma, renal cell carcinoma). Blockade of PD-1 restores the cytotoxic T-cell response (disinhibition), which promotes tumor cell apoptosis. Patients with advanced melanoma and other susceptible malignancies often have a drastic clinical response (eg, tumor regression, long-term remission) to PD-1 antagonists. In summary: Programmed-death receptor 1 (PD-1) is a checkpoint inhibitor that down regulates the cytotoxic T-cell response. Neoplastic cells often exploit this receptor via the over expression of PD-1 ligand. PD-1 receptor inhibitors (eg, Pembrolizumab) restore the T-cell response, allowing cytotoxic T cells to invade the tumor and induce apoptosis of neoplastic cells.

A 55-year-old woman comes to the office due to vaginal bleeding. The patient underwent menopause at age 49 but has had intermittent, light vaginal bleeding for the last 6 months. She has no pelvic pain and otherwise feels well. The patient has type 2 diabetes mellitus, which is managed with diet and exercise. Vital signs are normal. BMI is 38 kg/m2. Speculum examination shows dark red blood at the cervical os. On bimanual examination, the uterus is small and non tender and there are no palpable adnexal masses. Which of the following is the most likely underlying cause of this patient's symptoms?

Obese patient + postmenopausal bleeding most likely has endometrial hyperplasia or cancer. Common risk factor for endometrial hyperplasia/cancer is chronic unopposed estrogen exposure, associated with obesity, early menarche, late menopause, and chronic anovulation. Postmenopausal women have decreased ovarian function (ie, low estrogen and progesterone production), resulting in cessation of endometrial proliferation and subsequent amenorrhea. However, small amounts of androgens continue to be produced in the ovaries and adrenal glands after menopause. In obese women, these androgens (eg, androstenedione) are readily converted to estrogens (eg, estrone) by the aromatase enzyme found in adipose tissue. The increased peripheral production of estrogens in adipose tissue and the decrease in sex-hormone binding globulin levels (that normally occurs with menopause) lead to elevated free estrogen levels (Choice C). However, ovarian progesterone production remains low; therefore, the unopposed estrogen causes unregulated endometrial proliferation and subsequent endometrial hyperplasia or cancer, which often presents as postmenopausal or abnormal uterine bleeding.

A 78-year-old man with severe dementia is brought to the emergency department due to a daylong history of lethargy, fever, and vomiting. The patient lives in a nursing home, and his caregivers report that he has had poor oral intake over the last week. Medical history is significant for emphysema, heart failure with reduced ejection fraction, hypertension, and diabetes mellitus type 2. On physical examination, the patient is lethargic but arousable. Blood pressure is 60/30 mm Hg and pulse is 120/min and regular. The extremities are warm. Coarse rhonchi are heard over the right lower lung. After receiving several intravenous fluid boluses, the patient is given an intravenous infusion of an agent that increases peripheral vascular resistance, increases blood pressure, and decreases heart rate. Which of the following agents is most likely being described? A) dobutamine B) low dose epi C)high dose dobutamine D)phenylephrine

Phenylephrine is a selective alpha-1 agonist with no effect on alpha-2 or beta receptors. It causes peripheral vasoconstriction that increases systemic vascular resistance and blood pressure. It has no direct effect on the heart but stimulates a reflex-mediated decrease in heart rate and myocardial contractility.

A 32-year-old man undergoes evaluation for episodes of paroxysmal supraventricular tachycardia. An electrophysiology study reveals focal atrial tachycardia that originates in the crista terminalis of the right atrium. Radiofrequency ablation is planned to destroy the ectopic focus and treat the arrhythmia. Which of the following nerves is in close proximity to the treatment site and is at risk for injury during the ablation?

Phrenic nerve Paroxysmal supraventricular tachycardias (PSVTs) are arrhythmias that occur intermittently and have abrupt onset and offset. They arise from regions of abnormal electrical activity that occur in the atrioventricular node, an accessory pathway, or the atria. Radiofrequency ablation can be used to destroy the inciting cardiomyocytes and cure the arrhythmia. The crista terminalis, located in the right atrium, is a common site of origination for atrial PSVT. However, the right phrenic nerve, which innervates to the right hemidiaphragm, courses along the pericardium that overlies the right atrium and is at risk of injury during procedures that target structures in or near the right atrium (eg, crista terminalis, cavotricuspid isthmus, right-sided pulmonary veins). Such injury is typically recognized by elevation of the right hemidiaphragm on chest x-ray.

A 70-year-old man comes to the office due to 3 weeks of unrelenting low back pain. He was grocery shopping when he first noticed the pain. The patient reports no trauma or leg weakness but describes having to strain to urinate. He has a history of hypertension and hyperlipidemia and has not seen a doctor in the past 5 years. His wife died a year ago, and he now lives alone. The patient is a retired construction worker and has a history of tobacco and marijuana use. There is tenderness in the lower vertebral area. Imaging study of the spine reveals several osteoblastic lesions in the lumbar vertebrae. Which of the following structures was most likely involved during the spread of this patient's disease?

Prostate venous plexsus Prostate cancer with metastases to the bone. After regional lymph nodes, liver, and lungs, the skeletal system is the fourth most common site of metastases, which usually disseminate hematogenously. Cancers of the pelvis, including the prostate, spread to the lumbosacral spine via the vertebral venous plexus (VVP). The VVP communicates with a number of venous networks, including the prostatic venous plexus, which receives the venous supply from the prostate, penis, and bladder. It runs up the entire spinal column and connects with the venous supply of the brain via a valveless system, which allows for bidirectional flow and regulation of intracranial pressure. This venous connection to the cerebral circulation may help explain the propensity of tumors to metastasize to the brain.

A 24-year-old woman, gravida 1 para 1, comes to the office for an annual visit. The patient is up to date with cervical cancer screening and has completed the human papillomavirus vaccination series. She is in a monogamous relationship with her husband and has negative chlamydia and gonorrhea screening. The patient has been using a progestin-only pill since giving birth a year ago. She would like to switch to a combined oral contraceptive as she is no longer breastfeeding. Which of the following will be the primary mechanism of pregnancy prevention when the patient switches to a combined hormonal oral contraceptive?

Reduction in serum gonadotropin releasing hormone. Progestin, the synthetic equivalent to progesterone, is responsible for pregnancy prevention in all hormonal contraceptives. Estrogen is often included to improve the bleeding profile. The primary mechanism of action of a progestin depends on its pharmacokinetics (type of progestin, mode of administration, dose). Progestins that are more potent produce both systemic and local effects (levonorgestrel in pill form), whereas less potent progestins produce local uterine effects only (levonorgestrel released by the intrauterine device). Combined hormonal contraceptives exhibit systemic effects by suppressing GnRH in the hypothalamus, which decreases synthesis of the gonadotropins FSH and LH in the anterior pituitary. Because an LH spike is required to stimulate ovulation, ovulation is inhibited.

A 56-year-old man comes to the emergency department due to right lower extremity pain and swelling. The symptoms began shortly after he flew back to the United States from a 7-day cruise in Indonesia. Doppler ultrasonography reveals a femoropopliteal venous thrombosis, and the patient is given anticoagulation with rivaroxaban. Compared to vitamin K antagonists, this patient's treatment is most likely to have which of the following benefits?

Rivaroxaban is a direct oral anticoagulant (DOAC) that binds to and inhibits factor Xa, thereby preventing conversion of prothrombin to thrombin. DOACs are generally preferred over vitamin K antagonists (VKAs) such as warfarin for most conditions requiring prolonged anticoagulation due to less variability in therapeutic effect. The therapeutic effect of VKAs is affected by changes to dietary intake of vitamin K, disruption to vitamin K- producing intestinal flora (eg, antibiotics), and a wide range of medications that inhibit or induce cytochrome P450 2C9; therefore, patients given VKAs require coagulation monitoring (eg, with INR) to ensure adequate anticoagulation. In contrast, the therapeutic effect of DOACs is not typically altered by environmental or medication changes; therefore, laboratory monitoring is not required. DOACs may also have a lower risk of intracranial bleeding, the most serious risk of anticoagulation, when compared to VKAs.

A 43-year-old woman comes to the office due to acute back pain after dragging a heavy box. The pain is located in her lower back and radiates down the right posterior thigh to the foot. The patient describes the pain as "shooting" and grades it 8/10 in intensity. She has no bowel or bladder symptoms. The patient has tried over-the-counter analgesics with limited symptomatic relief. Vital signs are within normal limits. On physical examination, straight leg raise testing is positive on the right. Right hip extension is weaker when compared to the left. Knee jerk reflexes are 2+ and bilaterally symmetric, but the right ankle jerk reflex is absent. Which of the following nerve roots is most likely affected in this patient? 1) S1 2)L3 3)L4

S1 AND S2= BUCKLE MY SHOE ACHILIS TENDON L3-L4= KICK THE DOOR= PATELLAR REFLEX C5-C6= PICK UP THE STICKS= BICEPS C7-C8= TRICEPS

A 22-year-old woman, who recently relocated, comes to the office for a new patient visit. She has mild intellectual disability and has completed a high school level of education. The patient has no major health problems but reports persistent swelling of the hands and feet. Menarche occurred at age 13, and she has regular menstrual cycles. Physical examination shows short stature and a webbed neck. Karyotype analysis performed on peripheral leukocytes shows that 40% of the cells have a 45,X genotype and that the remaining 60% contain a 46,XX genotype. Which of the following is the most likely cause of this patient's condition? A) Germline mosaicism B)Somatic mosaicism C)Uniparental disomy D) Xlinked inactivation

Somatic mosaicism affects the cells forming the body(hense the karyotype on the leukocyte) , causing disease manifestations to develop in affected individuals. 45,X/46,XX is the most commonly diagnosed mosaicism affecting sex chromosomes. These patients typically have a milder form of Turner syndrome or can be asymptomatic, depending on the ratio of abnormal to normal cells.

A 20-year-old man is brought to the emergency department after a motor vehicle collision. Blood pressure is 130/84 mm Hg, pulse is 108/min, and respirations are 18/min. The airway is intact, and breath and heart sounds are normal. There is bruising across the central lower abdomen and the suprapubic area is tender to palpation. Chest x-ray and pelvic x-ray reveal no fractures. Bedside ultrasound shows intraperitoneal free fluid. Urine dipstick test is positive for blood. CT scan of the abdomen and pelvis is most likely to reveal which of the following injuries in this patient?

The bladder is a hollow pelvic organ located just posterior to the pubic symphysis. Although the bladder is extraperitoneal, the bladder dome is covered by peritoneal lining and extends into the peritoneal cavity when distended with urine. Blunt lower abdominal trauma can abruptly increase intravesicular pressure (especially when the bladder is full) and cause the bladder to rupture at the dome, where it is most distended and least supported by surrounding structures. As a result, urine is diverted from the urinary tract into the peritoneal cavity and can be seen on imaging as intraperitoneal free fluid. Peritonitis often does not develop acutely in these patients because urine is typically sterile.

A 16-year-old boy is being evaluated for difficulty walking. On physical examination, he has thoracic scoliosis and multiple small scars on his hands. When asked about these, the boy replies, "I frequently burn my hands by accident because I can't feel the heat." Neurologic examination reveals 1+ biceps reflexes and 3+ patellar reflexes bilaterally, as well as decreased muscle strength in the bilateral upper and lower extremities. His hand muscles are atrophic, and position and vibration senses in the feet are also impaired. Which of the following is the most likely cause of this patient's problems?

Syringomyelia This patient has chronic loss of upper extremity pain and temperature sensations, upper extremity weakness and hyporeflexia, lower extremity weakness and hyperreflexia, and kyphoscoliosis. These findings are consistent with syringomyelia. In syringomyelia, a central cystic dilation in the cervical spinal cord (a syrinx) slowly enlarges, characteristically causing damage to the ventral white commissure and anterior horns. The ventral white commissure is the site of decussation of second-order lateral spinothalamic tract neurons, and the ventral horns are the site of lower motor neuron cell bodies. The syrinx is most commonly situated at the C8-T1 cord levels and may extend rostrally, caudally, and centrifugally. Further expansion within the cervical cord in later stages of the disease can produce lower extremity weakness and hyperreflexia (an upper motor neuron defect) by affecting the lateral corticospinal tract and can also lead to loss of position and vibration senses in the feet due to involvement of the posterior columns. Scoliosis can occur due to paresis of paravertebral muscles. off note this is not ALS becuase that is caused by degeneration of UMN and LMN and there is no sensory loss.

A 44-year-old man comes to the emergency department with a 3-day history of fever, chills, malaise, dyspnea, and a cough productive of "greenish" sputum. The patient has no prior medical problems and has never been hospitalized. He has a 25-pack-year smoking history and drinks 4-5 beers a week. His temperature is 39.4 C (103 F), blood pressure is 130/80 mm Hg, pulse is 98/min, and respirations are 20/min. On examination, dullness to percussion, crackles, and egophony are present at the right lung base. The remainder of the examination is normal. Chest x-ray shows a dense infiltrate occupying the entire right lower lobe. Which of the following most likely accounts for the color of this patient's sputum? A) mucopolysaccrides B)myeloperoxidase C)High bacterial load D)necrosis

The green discoloration of pus or sputum seen during common bacterial infections is due to the presence of myeloperoxidase, a blue-green heme-based enzyme that is released from neutrophil azurophilic granules and forms hypochlorous acid (bleach).

A 58-year-old previously healthy woman comes to the emergency department due to left lower abdominal pain and fever. CT scan of the abdomen shows acute diverticulitis with microperforation. The patient is hospitalized and empiric antibiotics are begun. Food and fluids are withheld to promote bowel rest, and an isotonic saline infusion is administered for hydration. Over the next 2 days, the patient's fever and abdominal pain gradually improve, and pulse, blood pressure, and urine output remain within normal limits. Total administration of sodium chloride has been approximately 5 L. Compared with pre admission levels, which of the following changes have most likely occurred in this patient?

This patient likely has nonanion gap metabolic acidosis (NAGMA) due to infusion of excess normal saline. Excess sodium chloride increases serum chloride (Cl−) to cause hyperchloremia. Because Cl− and bicarbonate (HCO3−) are the predominant anions in the body, the increased serum Cl− causes intracellular shifting of HCO3− to maintain electronegative balance. This "loss" of HCO3− (reduced serum HCO3−) decreases blood pH. Infusion of excess normal saline also increases intravascular volume, which the kidneys respond to by increasing sodium (Na+) excretion, resulting in increased urine Na+.

A 78-year-old man comes to the office due to a one-month history of progressive dyspnea, generalized weakness, fatigue, and palpitations. He also reports tingling and numbness in both lower limbs. His daughter, who is visiting from another state, adds that since his wife's death a year ago, the patient has not been taking care of himself. Blood pressure is 105/50 mm Hg and pulse is 104/min. Cardiovascular examination shows a displaced apical impulse at the sixth intercostal space, a third heart sound, and high-volume, collapsing carotid pulses. Bilateral basal crackles, 2+ bilateral pedal edema, and mild hepatomegaly are also present. Neurologic examination shows decreased light touch and vibration sense in the feet, with decreased knee and ankle reflexes bilaterally. Laboratory evaluation shows normal blood counts. Deficiency of which of the following nutrients is most likely responsible for this patient's symptoms? A) cobalimine B) pyridoxine C) thiamine D) ascorbic acid

Thiamine deficiency causes beriberi and Wernicke-Korsakoff syndrome. Dry beriberi is characterized by symmetrical peripheral neuropathy; wet beriberi includes the addition of high-output congestive heart failure.

A 66-year-old man with diabetes mellitus is brought to the hospital due to sudden-onset chest pain and nausea. Blood pressure is 70/60 mm Hg and pulse is 60/min. Lungs are clear on auscultation. ECG shows ST-segment elevation in leads II, III, and aVF. Chest x-ray is unremarkable. The patient is diagnosed with an inferior wall myocardial infarction. Emergent cardiac catheterization reveals complete occlusion of the proximal right coronary artery. He is persistently hypotensive in the cardiac catheterization laboratory. Which of the following hemodynamic findings is most likely to be observed in this patient? CO, PCWP, AND Central venous pressures (increase, decrease or no change?) Findings?

This patient has an inferior wall MI and he has a complete occlusion of the right coronary artery, which also gives off marginal branches that supply most of the right ventricle (RV). Hence inferior wall MI are often associated with right ventricular infarction. Right-sided heart failure typically presents with hypotension and distended jugular veins, but the lungs are clear unless significant left-sided heart failure is also present. This patient's lack of crackles on lung auscultation and absence of pulmonary edema on chest x-ray is indicative of isolated RV dysfunction. Right ventricular infarction decreases RV stroke volume, which in turn leads to diminished LV filling and cardiac output despite preserved LV systolic function, resulting in hypotension and shock. Because left-sided filling pressures are reduced, pulmonary capillary wedge pressure (PCWP) also decreases as it reflects left atrial pressure. In addition, patients have elevated central venous pressure due to impaired forward flow and backup of blood into the systemic venous system.

A 34-year-old primigravida at 18 weeks gestation comes to the office for a routine prenatal examination. The patient's pregnancy has been uncomplicated. She takes a daily prenatal vitamin, and her prenatal laboratory results have been normal to date. The patient's personal and family medical histories are unremarkable. During the visit, a detailed fetal ultrasound reveals unilateral hydronephrosis. Male external genitalia are also visible. If the fetal hydronephrosis is caused by obstruction, which of the following is the most likely site?

Uretopelvic junction. T he most common pathologic cause of unilateral fetal hydronephrosis is a narrowing or kinking of the proximal ureter at the ureteropelvic junction (UPJ). -Embryologically, the fetal genitourinary tract is derived from the metanephric blastema and the ureteric bud (a dorsal outgrowth from the mesonephric duct). -The metanephric blastema gives rise to functioning renal parenchyma by 10 weeks gestation while the ureteric bud develops into the renal pelvis and ureters through dilation and canalization. The UPJ is the last segment of the fetal ureter to canalize. The pathogenesis of UPJ obstruction may involve failure of this canalization through abnormal development of circular musculature and/or collagen fibers. - Posterior urethral valves are the most common cause of bilateral fetal hydronephrosis in boys. They are caused by an obstructive, persistent urogenital membrane at the junction of the bladder and urethra. Other congenital anomalies that cause bilateral fetal hydronephrosis include urethral strictures, meatal stenosis, and bladder neck obstruction. Neurogenic causes of obstructive uropathy (eg, spinal cord damage) can also result in bilateral hydronephrosis.

A 44-year-old man comes to the office due to low mood, impaired concentration, increased sleep and appetite, feelings of heaviness in his arms and legs, and loss of energy. He is having difficulty at work as he is overly sensitive to criticism. Although his boss has told him not to worry, the patient is concerned that his job is in jeopardy due to poor performance. His symptoms started 8 months ago without any clear-cut precipitating event. The patient received adequate trials of 3 different antidepressants without improvement and has been off of medication for the past several weeks. His physician is now considering electroconvulsive therapy (ECT). The patient declines ECT and asks to try another medication. The physician then considers phenelzine. The presence of which of the following additional symptoms would make the patient more likely to respond to this medication? A) Auditory hallucinations B) Decreased libido C) delusions D) Mood reactivity E) Rapid speech

Atypical depression: MDD with atypical features is characterized by mood reactivity (ie, mood improves in response to positive events), leaden paralysis (ie, patient's arms and legs feel extremely heavy), rejection sensitivity (ie, overly sensitive to slight criticism), and the reversed vegetative signs of increased sleep and appetite. This patient is both treatment-resistant (failure of multiple antidepressant agents) and has atypical features, making him a good candidate for a trial of an MAOI.

A research scientist studying the metabolic pathways that contribute to obesity feeds experimental animals a high-carbohydrate, high-protein diet for a prolonged period. A sample of liver tissue is then obtained from the animals, and the activity of various enzymes involved in fatty acid metabolism is measured and recorded. It is determined that beta-oxidation of fatty acids is inhibited within these cells as a result of the diet. An increase in which of the following substances is most likely responsible for the observed effect? A) citrate B) Carnitine C)malonyl coA D) NADPH E)Acetoacetate

C Cytosolic acetyl-CoA carboxylase converts acetyl-CoA to malonyl-CoA during the rate-limiting step of de novo fatty acid synthesis. Malonyl-CoA also inhibits the action of mitochondrial carnitine acyltransferase, thereby inhibiting beta-oxidation of newly formed fatty acids.

A 63-year-old man collapses at home after experiencing severe retrosternal chest pain. The emergency medical team finds him with no pulse, and cardiac monitoring shows ventricular fibrillation. Cardiopulmonary resuscitation is performed, and the patient regains spontaneous circulation after several minutes. On arrival at the hospital, he has stable vital signs but does not follow commands or respond to strong stimuli. Which of the following areas is most likely to be affected first by ischemic damage in this patient?

Irreversible damage begins after 5 mins of hypoxia. Most vulnerable: Hippocampus, neocortex, cerebellum, watershed areas Cardiac arrest, secondary to ventricular fibrillation, caused rapid cessation of cerebral blood flow and global cerebral ischemia. Global ischemia lasting longer than 3-5 minutes can cause irreversible damage to neurons. The cells that are typically affected first by hypoxia are the CA1 pyramidal neurons of the hippocampus, which can be damaged by ischemia in as little as 3 minutes. The cerebellar Purkinje cells and neocortex pyramidal neurons are also highly susceptible to damage if ischemia lasts for 5-10 minutes. These neurons are likely vulnerable to ischemia because of their high metabolic demand and unique neurochemical characteristics (eg, inability to repolarize after anoxic depolarization). The hippocampus is located in the medial temporal lobe and plays an essential role in memory, spatial processing, and response inhibition. Selective damage to the hippocampus is common with transient ischemia and can result in an inability to make new memories (anterograde amnesia) and disorientation to place or time.

A 42-year-old woman is hospitalized for surgical removal of a mediastinal mass. The patient has had intermittent episodes of double vision, difficulty chewing food, and weakness of the extremities for the past several months. During evaluation she was found to have a circumscribed anterior mediastinal mass. Preoperative nerve conduction studies revealed a decremental response to repetitive motor nerve stimulation. She has no other medical conditions and does not use tobacco, alcohol, or illicit drugs. The patient has no drug allergies. Compared to a healthy individual, which of the following anesthesia-related medications is most likely to have increased potency in this patient?

Neuromuscular blocking agents are used to induce skeletal muscle relaxation during surgery by competing with acetylcholine at the postsynaptic membrane. They can be subclassified by their effects: Nondepolarizing agents (eg, vecuronium, rocuronium) are competitive antagonists of nicotinic acetylcholine receptors. Due to the low number of functioning receptors, patients with MG are extremely sensitive to these agents; very small doses can induce paralysis with loss of airway patency. Depolarizing agents (eg, succinylcholine) are competitive agonists of nicotinic acetylcholine receptors; persistent depolarization leads to desensitization of the motor end plate. Due to the low number of functioning receptors, patients with MG are often resistant to these agents; extremely large doses are required to induce muscle relaxation.

A 26-year-old man comes to the emergency department due to fever, right flank pain, and difficulty walking for the past 3 days. A week ago, the patient was treated with cephalexin for bacterial folliculitis. He was diagnosed with type I diabetes mellitus 10 years ago and takes subcutaneous insulin. Temperature is 38.9° C (102° F). During the medical interview, the patient lies supine on the examination table with his right hip and knee flexed and the limb externally rotated. On physical examination, he resists passive extension of the limb due to worsening of the pain. This patient's pathological process most likely involves which of the following muscles? A) quadratous lamborum B)obturator exturnus C)psoas major D) Quadriceps femoris

Psoas abscess presents with fever, back or flank pain, an inguinal mass, and difficulty walking. Inflammation of the psoas muscle leads to pain with extension at the hip (ie, psoas sign). Psoas abscess can develop due to hematogenous or lymphatic seeding from a distant site, particularly in patients with diabetes mellitus, intravenous drug use, and immunosuppression (eg, HIV infection).

A research scientist is studying biochemical reactions that take place in the liver. He cultures hepatocytes in a growth media enriched with glutamate labeled with nitrogen isotopes. After some time, he finds that the nitrogen isotopes are transferred to oxaloacetate, forming aspartate in the process. Which of the following substances is most likely involved in this reaction?

Pyridoxine (vitamin B6) is necessary for the transamination and decarboxylation of amino acids, for gluconeogenesis, and for other essential biochemical processes. Transamination reactions typically occur between an amino acid and an α-keto acid. The amino group is transferred to the α-keto acid from the amino acid, and the α-keto acid thereby becomes an amino acid. For example, glutamate (amino acid) reacts with oxaloacetate (α-keto acid) to form aspartate (the resulting amino acid) and α-ketoglutarate (the resulting α-keto acid). Transaminases (aminotransferases) are the enzymes that catalyze transamination reactions, and pyridoxal phosphate (active vitamin B6) serves as an essential cofactor for the transaminase. B6 kaplan notes: Aminotransferase reactions (protein metabolism) • Alanine aminotransferase (ALT) • Aspartate aminotransferase (AST) • Delta-aminolevulinate synthase ↑ in liver & muscle disease • Rate-limiting enzyme in heme synthesis • Deficiency• Isoniazid therapy (most common cause)• Clinical manifestations • Sideroblastic anemia • Cheilosis/stomatitis • Convulsions

Physicians involved in regenerative medicine research conduct a series of animal experiments to determine pulmonary tissue regeneration capacity. During one of the experiments, lung alveoli are exposed to NO2 and massive necrosis of the epithelial lining ensues. Histologic examination of the injured tissues a month later shows partial recovery of the alveolar epithelial lining. This regenerated tissue is most likely derived from which of the following cell types?

Type 2 pneumocyte produce surfactant and act as progenitor cells for type 1 pneumocyte ninety-five percent of the alveolar surface is covered by flat type I pneumocytes. The remaining 5% is covered by interspersed cuboidal type II pneumocytes that make up more than half of the total number of alveolar cells. Type II pneumocytes are the source of pulmonary surfactant and have the ability to proliferate in response to injury. They also act as progenitor cells for type I pneumocytes, which cannot regenerate on their own (Choice E).

A 56-year-old man is evaluated for dyspnea and chest discomfort. Medical history includes hypertension, type 2 diabetes mellitus, and chronic kidney disease. Temperature is 37.8 C (100 F), blood pressure is 160/90 mm Hg, pulse is 98/min, and respirations are 24/min. BMI is 31 kg/m2. The patient undergoes a 2-part nuclear medicine test. During the first part, he breathes in a radioactive tracer and a lung scan is performed. In the second part of the test, he is given an intravenous injection of a different radioactive material while breathing normal air, and the scan is repeated. When the images are compared, a large area of the lower right lung is not visualized in the second scan. Based on the observed findings, which of the following underlying disease processes is most likely present in this patient?' A) DVT B) Pulmonary congestion and pleural effusion c)Spontaneous pneumothorax

Ventilation-perfusion (V/Q) scans use radiotracers to compare the ventilation and blood perfusion of each area of the lung. V/Q mismatch with perfusion defects are often indicative of a pulmonary embolism, which are most commonly caused by deep vein thrombosis in the lower extremities that embolizes to the pulmonary vasculature.

A 62-year-old man comes to the physician due to intermittent groin pain. The pain is most severe when the patient lifts heavy loads and after a long day at his job as a construction worker. Physical examination shows a right-sided groin bulge directly above the inguinal ligament. The bulge increases in size when he bears down. An ultrasound reveals that the mass originates medial to the inferior epigastric vessels. This patient's condition is most likely caused by which of the following?

Weakness of transversalis fascia Direct inguinal hernias occur most commonly in older men due to weakness of the transversalis fascia. They protrude medial to the inferior epigastric vessels into the Hesselbach triangle and pass only through the superficial inguinal ring with no direct route to the scrotum.

A 67-year-old woman comes to the office due to difficulty concentrating. She reports repeatedly reading the same page of material due to an inability to focus. She also describes memory problems, stating that it "takes a while to remember another person's name." The patient's medical history is significant for hypertension and type 2 diabetes mellitus. These are well controlled through diet, exercise, and oral medications. She lives alone and describes her mood as "good." Blood pressure is 127/67 mm Hg and pulse is 65/min. The physician decides to administer a brief cognitive test to assess her difficulty in concentrating. Which of the following elements of cognitive testing would best assess for impaired attention and concentration in this patient? A) Recalling three unrelated words after 5 mins b) Reciting months of the year backwards C) Remembering a past event D) writing a complete sentence e) following a 3 step command

b A- short term memory c) long term memory- remote memory d-language e- comprehension executive function is like drawing a clock

A 14-year-old girl is brought to the office by her mother due to unusual behavior over the past year. The mother is concerned that her daughter seems overly anxious about eating, and says, "During lunch, she spent 45 minutes cutting the broccoli on her plate until all the pieces were the same size." The patient recently became a vegetarian and spends hours in the grocery store selecting vegetables and checking nutritional information. She also seems very concerned about her appearance; she brushes her hair 21 times on each side, parts her hair strand by strand, and plucks her eyebrows for an hour daily. The patient says, "If I don't keep things exactly even I'm afraid something bad might happen." The patient is medically healthy and has not yet started menstruating. She weighs 45.8 kg (101 lb) and is 157.5 cm (5 ft 2 in) tall. BMI is 18.5 kg/m2. Physical examination is normal with the exception of extremely sparse eyebrows. Which of the following is the most likely primary diagnosis? A) OCD B) obsessive compulsive personality disorder

A OCD is egodystonic OCPD is ego syntonic Obsessive-compulsive personality disorder involves a lifelong pattern of insistence on control, orderliness, and perfection and does not include frank obsessions and compulsions that are present in OCD. Personality disorders are generally not diagnosed before age 18 (or in young adolescents) and would not explain this patient's recurrent compulsions performed in response to anxiety-inducing obsessions.

A 5-year-old girl is brought to the clinic due to 2 days of dark red urine. The patient was treated for facial impetigo 1 month ago. Blood pressure is 140/90 mm Hg. Urinalysis reveals hematuria, mild proteinuria, and occasional red blood cell casts. Which of the following changes would most likely be present on this patient's renal biopsy? A) Diffuse capillary thickening on light microscopy B)Discrete subepithelial humps on electron microscopy C)Glomerular basement membrane fibrin deposition EM D) GBM splitting on LM E) Linear IgG and C3 deposits on IF

B Poststreptococcal glomerulonephritis is an immune complex-deposition disease that occurs 2-4 weeks after exposure to group A beta-hemolytic Streptococcus species (eg, pharyngitis, skin infection). Light microscopy shows enlarged, hypercellular glomeruli. Immunofluorescence demonstrates a "lumpy-bumpy" granular deposits of IgG and C3 on the glomerular basement membrane, and subepithelial, electron-dense deposits are seen on electron microscopy.

A 38-year-old woman comes to the emergency department due to severe headache. An hour ago, she experienced a sudden, explosive headache followed by a brief loss of consciousness. Since regaining consciousness spontaneously, she has had persistent global headache and nausea. Head movement or bright light worsens the pain. Temperature is 37.2 C (99 F), blood pressure is 142/90 mm Hg, and pulse is 98/min. The patient is somnolent but easily rouses to voice. She is well oriented to time, place, and person. There is increased resistance to neck flexion, which elicits pain. The pupils are equal and reactive to light. Deep tendon reflexes and sensory examination are normal. Brain imaging would most likely reveal which of the following? Dx?

Blood accumalation in the cerebral sulci This patient with a sudden-onset, explosive headache (ie, thunderclap headache), brief loss of consciousness, and signs of meningeal irritation (eg, neck stiffness, pain) likely has a subarachnoid hemorrhage (SAH). SAH usually results from the spontaneous rupture of a saccular (berry) aneurysm. These aneurysms most commonly develop at branch points of the anterior, middle, or posterior cerebral arteries in patients with a genetic predisposition (eg, family history, autosomal dominant polycystic kidney disease, Ehlers-Danlos syndrome). Blood empties into the subarachnoid space and pools in the cerebral sulci and basal cisterns, which is the diagnostic finding of SAH on CT scan. Blood in direct contact with the pia mater triggers severe headache and meningeal irritation. Nausea, photophobia, and transient loss of consciousness are also common. Many patients appear neurologically intact at presentation; however, progressively deteriorating mental status (eg, somnolence) is typical. In addition, SAH commonly triggers delayed cerebral vasospasm (eg, 4 days after initial bleeding) resulting in stroke with focal neurologic deficits.

A 6-month-old boy is brought to the clinic by his mother and grandmother for a routine well-child visit. The child was born full-term and has no medical problems. He recently learned to sit with support and is starting to eat pureed foods. Vital signs and physical examination are normal. As part of the routine pediatric immunization schedule, the pneumococcal conjugate vaccine is ordered. The patient's grandmother says that she recently received the pneumococcal polysaccharide vaccine. Which of the following statements is true regarding the difference between the pneumococcal conjugate and polysaccharide vaccines?

Conjugate vaccine induces a more robust immune response through B and T cell activation The 2 types of pneumococcal vaccines are the polysaccharide and conjugate vaccines. The 23-valent pneumococcal polysaccharide vaccine (PPSV23, Pneumovax) protects against a wider range of serotypes but antibody levels decline over approximately 5 years. In addition, PPSV23 is not immunogenic in children age <2 due to their relatively immature humoral antibody response. Therefore, PPSV23 is recommended for all adults age >65 and for those age 2-64 with certain medical conditions (eg, diabetes, chronic pulmonary or cardiovascular disease). The 13-valent pneumococcal conjugate vaccine (PCV13, Prevnar) contains a nontoxic diphtheria protein conjugated to the polysaccharides that boosts the immune response through T cell recruitment. This conjugation allows for development of memory B cells, higher and longer-lasting antibody levels, and less mucosal carriage (herd immunity). Therefore, PCV13 is strongly immunogenic in infancy and part of routine childhood vaccinations. Similar to PPSV23, PCV13 is recommended for immunocompromised patients and adults age >65.

A 55-year-old man with poorly controlled hypertension and type 2 diabetes mellitus comes to the emergency department due to difficulty seeing. He has had blurry vision for the last 2 weeks, and 1 day ago he suddenly noticed a shadow develop across the visual field of his left eye. The patient has no recent headache, double vision, vertigo, light sensitivity, or nausea. On examination, pupils are equal and reactive to light. There is an area of reduced vision in the left eye. Funduscopic examination reveals a flame-shaped hemorrhage in the left temporal hemiretina. The transmission of visual information through which of the following neural structures will be disrupted?

Damage to the left temporal hemiretina will disrupt the transmission of visual information along the ipsilateral optic nerve, lateral optic chiasm, optic tract, lateral geniculate body, optic radiations, and primary visual cortex

A 55-year-old with woman with stage IV chronic kidney disease due to type 2 diabetes mellitus comes to the office for a follow-up visit. Blood pressure is 140/90 mm Hg and pulse is 78/min. BMI is 31 kg/m2. Laboratory results are as follows: Hemoglobin10.5 g/dL Calcium8.8 mg/dL Albumin3.7 g/dL Phosphorus7.2 mg/dL Creatinine3.3 mg/dL Blood urea nitrogen 88 mg/dL Parathyroid hormone 100 pg/mL (normal: 10-65) The patient's serum phosphorus has been persistently elevated despite strict dietary phosphate restriction. Treatment with sevelamer is initiated. This medication reduces the serum phosphorus level by which of the following mechanisms?

Decreases phosphate absorption in the intestine. oral phosphate binders are usually needed if dietary restriction is not sufficient to lower phosphate levels. Phosphate binders can be calcium containing (eg, calcium carbonate/acetate) or non-calcium containing (eg, sevelamer, lanthanum). Sevelamer is a nonabsorbable anion-exchange resin that binds intestinal phosphate to reduce systemic absorption. The resulting complex is eliminated in the feces. Other answer choices in the question explanation: Parathyroid hormone induces internalization and destruction of type IIa sodium/phosphate cotransporters (NPT2) in the proximal renal tubule. NPT2 gene expression is downregulated by fibroblast growth factor 23. These processes lead to decreased transcellular transport (and therefore decreased reabsorption) of phosphate in the renal tubules.

A 15-year-old male suffers severe cardiomyopathy following an infective myocarditis from Coxsackie virus and is placed on the cardiac transplant list. Two weeks following his cardiac transplantation from a matched donor, he is suffering dyspnea on exertion. Extensive evaluation is undertaken in this patient including cardiac catheterization and endomyocardial biopsy. Which of the following findings is most consistent with acute graft rejection?

Dense infiltrate of mononuclear cells usually composed primarily of T-lymphocytes. Acute cellular rejection of a transplanted organ is mediated by host T-lymphocyte sensitization against graft (foreign) MHC antigens. Acute rejection is usually diagnosed before symptoms set in because of close surveillance of these patients, but symptoms consistent with progressive rejection include those found in heart failure resulting from systolic dysfunction such as dyspnea on exertion or paroxysmal nocturnal dyspnea.

A 62-year-old man comes to the office with his wife due to behavioral changes. The patient has become increasingly impulsive and irritable over the past 1-2 years. He recently lost his job after making inappropriate comments and touching female coworkers. The patient has also lost interest in his hobbies and appears withdrawn during social gatherings. His other medical problems are hypertension and hyperlipidemia. A thorough neuropsychological assessment is conducted. Inability to perform which of the following tasks would be most indicative of frontal lobe dysfunction?

Describing a diff between a watch and ruler The frontal lobes are involved in personality, language, motor functions, and executive functions (eg, abstraction). Frontal lobe function may be diagnosed by testing abstraction ability (eg, asking about the similarities between two related objects) on the mental status examination.

A 12-year-old boy has prolonged oral bleeding immediately after a tooth extraction. Despite several interventions, the bleeding persists for hours and stops only after desmopressin (DDAVP) administration. The patient has several dental caries due to excessive juice intake and inconsistent oral hygiene. He has no other medical problems and takes no medications. His father and paternal grandmother have also had excessive bleeding after dental procedures. Review of systems is positive for mild bruising on his legs. Which of the following is the most likely therapeutic mechanism of desmopressin for this patient's condition?

Increase in endothelial protein release

A 28-year-old man comes to the emergency department due to anxiety, weakness, dyspnea, and headaches. He is a graduate student in chemistry and was accidentally exposed to nitrites in his research laboratory. Physical examination is notable for cyanosis that is not improved by administration of supplemental oxygen via facemask. Analysis of a sample of arterial blood drawn from this patient prior to any oxygen supplementation is most likely to reveal normal values for which of the following?

PaO2 of the blood. Iron bound to heme is normally in the reduced ferrous (Fe2+) state. Nitrites cause poisoning by inducing the conversion of this heme iron to the oxidized ferric (Fe3+) state, leading to the formation of methemoglobin. With iron in the oxidized ferric state, methemoglobin is unable to bind oxygen. In addition, the affinity of any residual ferrous iron in the hemoglobin tetramer is increased, causing a leftward shift of the oxygen-dissociation curve. However, the partial pressure of oxygen in blood, which represents the amount of oxygen dissolved in the plasma, is unchanged. Methemoglobinemia causes dusky discoloration of the skin (similar to cyanosis), and because methemoglobin is unable to carry oxygen, a state of functional anemia is induced.

A 28-year-old man comes to the office with bilateral skin lesions over the posterior surface of the elbows. The lesions are itchy at times; topical moisturizers have reduced the itching, but the lesions have remained. The patient has no chronic medical problems and takes no medications. Biopsy of the lesions demonstrates hyperkeratosis, parakeratosis, dilated capillaries in the dermal papillae, and foci of neutrophils in the superficial epidermis. Which of the following is the most likely diagnosis? A) psoriasis B) erythema nodosum C) atopic dermatitis D) dermatitis herpetiformis

Psoriasis is a common chronic inflammatory skin disorder that presents with scaly, erythematous plaques, typically on the extensor surfaces of the elbows and knees. In predisposed individuals, disruption of the epithelial barrier leads to activation of antigen-presenting dendritic cells and subsequently to a self-reinforcing inflammatory cascade characterized by recruitment and activation of T helper cells and proliferation of keratinocytes. Histopathologically, psoriasis is characterized by: Hyperkeratosis (thickening of the stratum corneum) and confluent parakeratosis (retention of nuclei in the stratum corneum) produces the characteristic silvery scale on gross examination Epidermal hyperplasia (acanthosis) with elongated rete ridges correspond to the typical erythematous plaques Neutrophilic foci in the stratum corneum and epidermis, which may coalesce to form microabscesses (Munro microabscesses) In addition, the epidermal cell layer superficial to the dermal papillae may be thinned and contain dilated blood vessels, which leads to pinpoint bleeding when the scale is removed from the plaque (Auspitz sign).

A 54-year-old previously healthy man comes to the office due to several weeks of leg swelling. He has had no fever, chest pain, or dyspnea. The patient has a 40-pack-year smoking history but does not use alcohol or illicit drugs. He is afebrile and vital signs are within normal limits. On physical examination, there is symmetric pitting edema of the lower extremities bilaterally. The abdomen is soft and nondistended. A mobile left flank mass can be palpated. There are several vertically oriented tortuous veins on the lower abdominal wall. Which of the following structures is most likely obstructed in this patient? A) femoral vein B) iliac vein C)saphenous vein D)IVC

Renal cell carcinoma tends to invade the renal vein; inferior vena cava obstruction can occur by intraluminal extension of the tumor. Obstruction of the inferior vena cava produces symmetric bilateral lower extremity edema, often associated with prominent development of venous collaterals in the abdominal wall.

A 29-year-old man is evaluated for 2 weeks of skin rash involving his hands, feet, and scalp. The rash is mildly pruritic and has progressively worsened. The patient has a history of HIV and is not adherent with antiretroviral therapy. Skin examination shows erythematous patches with scales and crusting. Biopsy of the rash shows organisms burrowed in the epidermis, as shown below: Look up the diagnosis in the Uworld flash card Test Id: 180733482

Scabies is a human mite infection associated with a pruritic papular rash with excoriations and burrows. Patients with impaired cell-mediated immunity (eg, HIV) often develop a very high mite burden. Treatment with topical permethrin and/or ivermectin is generally curative. Treatment is required to prevent discomfort, transmission, and potential complications (eg, secondary bacterial infection). First-line therapy includes topical permethrin, which blocks mite neurotransmission by impairing voltage-gated sodium channels. Permethrin cream is applied from the neck to the soles of the feet and left on for 8-14 hours. Oral ivermectin, an antiparasitic agent that binds chloride ion channels in invertebrate nerve and muscle cells, is an alternate medication for classic scabies and is used with permethrin for crusted scabies.

A 57-year-old man comes to the office due to 2 months of progressive cough and weight loss. He has a history of diabetes mellitus type 2 and chronic kidney disease. He does not smoke cigarettes. The patient traveled to Denmark 4 months ago for a 2-week vacation. He has 2 cats and a bird. He works in the maintenance department of a large hotel and services the hot tub and swimming pools. Temperature is 38 C (100.4 F), blood pressure is 140/100 mm Hg, pulse is 88/min, and respirations are 16/min. The patient is 190 cm (6 foot 3 inches) tall. BMI is 22 kg/m2. Examination shows cervical lymphadenopathy. Chest imaging is shown in the exhibit. Hemoglobin A1c is 8.5% and serum creatinine is 3.8 mg/dL. Which of the following risk factors most likely contributed to this patient's current condition?

The answer: underlying medical condition. Mycobacterium tuberculosis infection is controlled by coordination between macrophages and T lymphocytes. Conditions that impair immune function increase the risk of active tuberculosis; this includes chronic kidney disease, diabetes mellitus, HIV, substance abuse, malnutrition, and advanced age.

A 15-month-old boy is brought to the office by his parents for poor weight gain. The parents have also noticed frequent, large, foul-smelling stools. He was admitted to the hospital for a prolonged bout of bacterial pneumonia 6 months ago. Both parents are healthy and have no similar symptoms. The child was born at term at home. He is at the 20th percentile for length and 3rd percentile for weight. Vital signs are normal. On examination, the child appears thin and has a nontender, nondistended abdomen. Which of the following is most likely present in this patient?

This kid has CF. CF is an autosomal recessive disorder in which a defective chloride channel causes the accumulation of thick, dehydrated secretions in multiple organs. In the pancreas, viscous mucus obstructs small ducts and acini, preventing digestive enzymes from reaching the intestinal lumen and leading to progressive pancreatic inflammation and fibrosis. This ultimately causes pancreatic insufficiency, which may be present at birth in patients with CF and typically progresses throughout childhood. Signs of fat and protein malabsorption due to pancreatic insufficiency include steatorrhea (bulky, foul-smelling stools), failure to thrive, and fat-soluble vitamin deficiencies (A, D, E, and K). Elastase is a pro-enzyme (ie, zymogen) produced by pancreatic acinar cells and activated by trypsin within the duodenal lumen. Because elastase production and secretion are decreased in patients with pancreatic insufficiency, fecal elastase levels are also decreased; the presence of low fecal elastase is an accurate and noninvasive method for diagnosing pancreatic insufficiency of any cause (eg, CF, chronic pancreatitis).

A 6-month-old boy is brought to the office by his mother out of concern that he is not developing normally. He has been feeding regularly and has had no medical problems other than a mild respiratory infection a month earlier. However, the mother says, "he doesn't seem to be as interactive as my other children were at his age." Physical examination reveals delayed developmental milestones and hypotonia. Two years later, the child is found to have involuntary movements and demonstrates a tendency to aggressively bite his own lips and fingers. Laboratory analysis shows an elevated blood uric acid level. Activity of which of the following enzymes is most likely increased as a result of this patient's condition?

This patient has Lesch-Nyhan syndrome, an X-linked recessive disorder characterized by HYPERURICEMIA GOUT PISSED OFF RETARDED dysTonia. The condition is caused by deficiency of hypoxanthine-guanine phosphoribosyltransferase (HGPRT), an enzyme that normally functions in the purine salvage pathway to convert hypoxanthine back into IMP and guanine back into GMP. -The absence of HGPRT results in increased degradation of guanine and hypoxanthine bases into uric acid, which increases the demand for de novo purine synthesis. -The first step of purine synthesis is the formation of phosphoribosyl pyrophosphate (PRPP) by PRPP synthetase. PRPP can be used by adenine phosphoribosyltransferase and HGPRT for purine salvage, or it can be converted to phosphoribosylamine by PRPP amidotransferase in the first committed step of de novo purine synthesis. Because purine salvage is impaired in Lesch-Nyhan syndrome, the activity of PRPP amidotransferase must increase to supply a sufficient quantity of purine nucleotides.

A 5-year-old boy is brought to the emergency department due to an hour of epistaxis after he was elbowed in the face during a basketball game. He has a history of prolonged bleeding from the gums after brushing his teeth. Vital signs are normal. The patient has scattered bruises along his arms and legs. He receives desmopressin, which stops the bleeding. This treatment most likely improved the patient's condition through which of the following therapeutic mechanisms?

This patient has Von willibrand disease. VWD is caused by qualitative or quantitative defect in von Willebrand factor (vWF), a multimeric glycoprotein synthesized by endothelial cells and platelets. VWF contributes to hemostasis by forming a bridge between platelets and exposed subendothelial components at sites of injury and by serving as a carrier protein for factor VIII (whose half-life is tremendously shortened when not bound to vWF). Treatment of minor bleeding episodes in patients with vWD often involves desmopressin (DDAVP), a synthetic analogue of antidiuretic hormone that does not induce vasoconstriction. DDAVP induces a rapid and transient increase in vWF and factor VIII by indirectly releasing these compounds from endothelial storage sites. DDAVP is most useful in the acute setting, as repeated doses have limited effectiveness due to tachyphylaxis (endothelial stores become depleted).

A 65-year-old man is referred to a gastroenterology clinic by his primary care provider after he develops symptoms suspicious for colorectal cancer. The patient has a history of obesity, diabetes mellitus, and chronic obstructive pulmonary disease. He often eats steaks and hamburgers but avoids vegetables as "they have no flavor." The patient has smoked a pack of cigarettes daily for the past 40 years and occasionally consumes alcohol. He undergoes a colonoscopy and is found to have a mass lesion in the ascending colon. The mass is biopsied, and the histopathologic findings are consistent with colon adenocarcinoma. This patient most likely initially presented with which of the following clinical symptoms? A) abd pain, distention and nausea B) constipation alternating with diarrhea C) bloody stool and low grade fever D)Tenesmus and small caliber stool E) fatigue, and weight loss

This patient has a right sided lesion which usually present with iron def anemia. Remember that the ascending colon diameter is bigger than descending colon so its unlikely that youll have obstruction there. The location of colon adenocarcinoma can influence its clinical manifestations. Right-sided colon cancers usually grow as exophytic masses. Patients generally do not develop intestinal obstruction because the ascending colon has a larger caliber lumen than the descending colon and stool in the proximal colon is more liquid. Right-sided colon cancers typically present with features of iron deficiency anemia (eg, fatigue, pallor) due to occult blood loss. Nonspecific symptoms such as anorexia, malaise, and unintentional weight loss may also occur. Rectosigmoid involvement often causes hematochezia. Environmental and lifestyle factors that may have increased this patient's risk of colon cancer include obesity, cigarette smoking, consumption of red/processed meat, and low dietary intake of vegetables.

A 55-year-old man comes to the emergency department due to sudden-onset dyspnea. Medical history is significant for hypertension, hyperlipidemia, type 2 diabetes mellitus, and chronic kidney disease. The patient takes multiple medications and has no drug allergies. He works for an international bank and returned from a business trip in Australia a day ago. Blood pressure is 110/70 mm Hg and pulse is 110/min. Physical examination shows a moderately overweight man with tachypnea. The lungs are clear on auscultation. ECG shows sinus tachycardia. Ventilation/perfusion scanning is ordered. Which of the following findings would help confirm the suspected diagnosis in this patient? DX?

This patient has loads of co morbidities but one thing that sticks out in this question is his recent long travel. This patient most likely has a pulmonary embolism which causes perfusion defect without a ventilation defect. CT angiography is usually diagnostic in most patients but since he has kidney problems we cant give him the contrast as it can cause contrast induced nephropathy

An 8-month-old boy is evaluated for developmental delay, failure to thrive, and episodic seizures. Physical examination shows ophthalmoplegia and hypotonia. Laboratory studies reveal an elevated serum lactate level. Further histochemical studies show severely reduced pyruvate dehydrogenase enzyme activity in both freshly isolated peripheral blood lymphocytes and cultured fibroblasts. Increasing which of the following substances in his diet is most likely to help this patient generate energy without further elevating lactate levels?

This patient has pyruvate dehydrogenase deficiency. Recall that PDH is an irreversible enzyme which converts Pyruvate to acetyl coA. It requires cofactors: 1) Thymine pyrophosphate (B1) 2) Lipoic acid 3) Coenzyme A 4) FAD from (riboflavin B2) 5) NAD from (Niacin B3) Activators: insulin, High ADP, Ca levels and increase NAD/NADH ratio often presents with lactic acidosis and neurological defects. The substrates that accumulate are pyruvate and alanine. Treatment: Diet limited to ketogenic nutrients, high in fat content and rich in Lysine and leucine

A 17-year-old boy comes to the office due to a facial lesion. The patient noticed a small area of rough skin on his lower lip several weeks ago, which has progressively enlarged. He reports no pain or itching. The patient had eczema during early childhood but has had no other chronic medical conditions. Vital signs are within normal limits. Physical examination findings are shown in the exhibit. There are no other skin rashes, oral lesions, or enlarged lymph nodes. Which of the following histopathological findings are most likely present in this patient's skin lesion?

This patient has verucca vulgaris which is caused by HPV also known as common wart. Key findings on magnified view (ie, dermoscopy) include a roughened surface and small black dots representing thrombosed capillaries. Biopsy is usually not needed but can confirm the diagnosis in atypical cases. Histopathologic findings include: Epidermal hyperplasia (acanthosis) with eosinophilic inclusions and keratohyaline granules Thickened stratum corneum (hyperkeratosis) Papilloma formation Perinuclear cytoplasmic vacuolization (koilocytosis)

A 66-year-old woman is brought to the emergency department due to left leg pain. Five days ago, the patient underwent elective right hip arthroplasty for advanced osteoarthritis. She had been recovering well at an acute rehabilitation facility until she woke today with severe left calf pain and swelling. The patient has taken daily subcutaneous, low-molecular-weight heparin and as-needed acetaminophen since the surgery. She has no other medical conditions. Temperature is 37 C (98.6 F), blood pressure is 130/70 mm Hg, and pulse is 92/min. The surgical incision over her right hip is healing well. Her left posterior calf is tender and swollen to the knee. Three days ago, complete blood count was normal, but platelet count is now 70,000/mm3. Peripheral blood smear shows decreased platelet number but no other abnormalities. Which of the following is the most likely cause of this patient's thrombocytopenia? A) non immune mediated platelet clumping B)Platelet removal by splenic macrophage

This patient on low-molecular-weight heparin developed thrombocytopenia and deep venous thrombosis (calf pain/swelling), raising strong suspicion for heparin-induced thrombocytopenia (HIT) type 2. In HIT type 2, heparin induces a conformational change in platelet-factor 4, leading to the formation of a neoantigen. IgG antibodies form against the neoantigen (heparin-platelet factor 4), which results in antibody aggregation on the platelet surface. Patients generally develop manifestations 5-10 days after heparin initiation. Thrombocytopenia is the hallmark feature due to the clearance of antibody-coated platelets by splenic macrophages. Antibody aggregation on the platelet surface also results in wide-spread platelet aggregation, which worsens the thrombocytopenia and significantly increases the risk of venous and arterial thrombosis. Patients with HIT-type 2 should immediately stop heparin and be initiated on a nonheparin anticoagulant (eg, argatroban) to help prevent/treat thrombosis.

A 31-year-old man comes to the emergency department after a witnessed tonic-clonic seizure. The patient has no known medical problems. He grew up in a rural part of Guatemala and immigrated to the United States 3 years ago. He works as an architect and owns a cat. He has no history of tuberculosis. On examination, he is afebrile and has no focal neurologic deficits or meningeal signs. An HIV antibody test and an interferon-gamma release assay test are negative. Chest x-ray is normal. An MRI of the brain reveals a 2.5-centimeter cyst within the left sylvian fissure that has minimal enhancement and no associated edema. Which of the following is the most likely means of acquisition of the infection responsible for this patient's findings? A) handling of cat feces B) scratch from a cat C) inhalation of fungal spore D)Exposure to infected stool

This patient who grew up in a rural part of Guatemala and has no underlying immunocompromising conditions had a seizure and is found to have a cystic brain lesion on MRI. This presentation is most consistent with neurocysticercosis caused by Taenia solium, the pork tapeworm. Although rare in the United States, T solium is endemic in Central and South America (particularly in rural areas), and neurocysticercosis should be considered in patients from these areas who develop seizures or neurologic symptoms. Ingestion of eggs from the stool of tapeworm carriers can lead to cysticercosis (consumption of infected uncooked pork, which contains the larval cysts but not the eggs, can lead to tapeworm infestation [taeniasis] but not cysticercosis). Embryos attach to and penetrate the wall of the small intestine and subsequently migrate hematogenously to tissues, including the brain, where they form cysticerci, fluid-filled larval cysts surrounding the invaginated scolex (head of tapeworm with hooklets).

A 34-year-old woman comes to the office due to progressive exertional dyspnea for the past 6 months. She has no chest pain, lightheadedness, or syncope. Medical history is unremarkable and the patient takes no medications. She does not use tobacco, alcohol, or illicit drugs. The patient's mother died of pulmonary arterial hypertension at age 32. Blood pressure is 125/74 mm Hg and pulse is 75/min. BMI is 23 kg/m2. On physical examination, breath sounds are clear without wheezes or crackles. There is a loud S2 at the left upper sternal border. Chest x-ray reveals clear lungs. ECG shows right axis deviation. If this patient's condition is inherited, which of the following is the most likely direct cause of her disease? A) Right ventricular hypertrophy B) hypoxic vasoconstriction C) Thrombotic occlusion of pulmonary artery D) vasular smooth muscle proliferation

This patient's presentation and family history are suggestive of pulmonary arterial hypertension (PAH). Hereditary PAH is most often due to an inactivating mutation in BMPR2, which is transmitted in an autosomal dominant manner with variable penetrance. Patients with this mutation have a predisposition for dysfunctional endothelial and smooth muscle cell proliferation. An insult (eg, infection, drugs) is thought to then activate the disease process, which involves increased levels of vasoconstrictive, proliferative mediators (eg, endothelin) and decreased levels of vasodilative, antiproliferative mediators (eg, nitric oxide, prostacyclin). The end result is vasoconstriction with vascular smooth muscle proliferation, intimal thickening and fibrosis, increased pulmonary vascular resistance, and progressive pulmonary hypertension.

A scientist is conducting research to discover novel drug targets for antiretroviral agents against HIV. An HIV-positive blood sample is analyzed in the laboratory, and purified CD4 T lymphocytes are extracted. Using molecular tracing techniques, the scientist follows a viral HIV protein that is synthesized in the blood sample's T cells. The protein becomes glycosylated before being proteolytically cleaved into 2 smaller proteins in the Golgi apparatus. Which of the following best describes the function of these 2 newly formed HIV proteins?

Virion attachement to the target cell. The HIV genome contains a set of structural genes (gag, pol, and env) that encode polyproteins. These polyproteins are subsequently cleaved by host or viral proteases into the individual proteins that compose the HIV virus. The only glycosylated HIV polyprotein is gp160, the product of the env gene. gp160 is extensively glycosylated in the rough endoplasmic reticulum and Golgi apparatus. It is then cleaved into the envelope proteins gp120, which mediates viral attachment, and gp41, which mediates viral fusion with the host cell. Glycosylation of these proteins is crucial for: Immune evasion - Glycosylation acts as a shield for the conserved regions of these surface proteins, masking them from immune recognition. Host cell binding Proper protein folding

A 35-year-old woman comes to the office due to insomnia and fatigue over the past month. She has felt increasingly depressed,irritable, and worthless since being let go from her job 5 weeks ago. The week prior to her last menstrual period was particularly difficult, and she stayed in bed most of the day. Over the last month, the patient has lost 3.6 kg (8 lb) and has felt unmotivated and unable to concentrate on looking for new work. Other medical conditions include hypothyroidism and migraine headaches. The patient drinks 1 or 2 glasses of wine a few times a week and smokes marijuana once a month. Medications include levothyroxine and naproxen. Physical examination is normal. TSH level is 0.9 µU/mL. Which of the following is the most likely diagnosis in this patient?

major depressive disorder. This patient's symptoms (eg, depressed mood, feelings of worthlessness, low energy, insomnia, weight loss, and poor concentration) lasting ≥2 weeks meets the criteria for major depressive disorder (MDD). The presence of MDD criterion symptoms for ≥2 weeks is all that is required for diagnosis, but the median duration of an MDD episode is approximately 6 months. Accurate diagnosis of MDD requires ruling out medical and substance-induced causes. Despite a history of hypothyroidism in this patient, her normal TSH level indicates that her hypothyroidism is adequately controlled with levothyroxine and would not be considered a medical cause of her depression (Choice C). Patients with sufficient depressive symptoms are diagnosed with MDD even if there is a clear psychosocial stressor that precipitated the depression (ie, this patient meets the criteria for MDD and therefore cannot be diagnosed with an adjustment disorder) (Choice A). In patients who do not meet the criteria for MDD, adjustment disorder with depressed mood may be diagnosed if sadness, distress, and functional impairment develop ≤3 months following a psychosocial stressor.

A surveillance study is conducted to assess the efficacy and safety of tolvaptan, a drug currently being used in the clinical setting for treating heart failure with volume overload. The study enrolls 8,300 patients with a variety of medical comorbidities who received tolvaptan once daily in the morning for 2 weeks. Results showed that tolvaptan demonstrated aquaretic efficacy in patients with diuretic-resistant volume overload, but that hypernatremia was a complication in a small number of patients. Researchers recommended a lower dose of tolvaptan in those with normonatremia and hypokalemia to prevent hypernatremia. Which of the following best describes this type of study? A) pre clinical B) phase 1 C) phase 2 D) phase 3 E) phase 4

phase 4 Phase IV trials are postmarketing surveillance studies that assess for rare or delayed adverse effects of newly approved treatments on the market. They are conducted on larger patient populations for longer periods of time compared to other types of clinical trials. As such, they have substantial statistical power and can identify adverse effects that were not observed in earlier trials; they may also evaluate long-term effectiveness of new treatments and assess treatment response in different subpopulations (eg, based on sex, comorbidities). Results from phase IV trials may lead to restrictions in use or a withdrawal of treatment from the market. In this example, a postmarketing surveillance study was conducted to determine if the efficacy and safety features established in controlled trials for a treatment (ie, tolvaptan) in a particular population (ie, heart failure subjects with volume overload) extend to real-world clinical settings. The study revealed a treatment complication in the clinical setting (ie, hypernatremia) that warranted a use restriction (ie, lower dose) in a specific population (ie, those with normonatremia and hypokalemia). This study describes a phase IV trial.


Conjuntos de estudio relacionados

Before conception and Pregnancy (14+15)

View Set

Chapter 10: Interest Groups (Inquizitive)

View Set

Partnerships & LLCs Knowledge Check

View Set

Week 4: Chapter 4: The Time Value of Money- Stream of Cash flows

View Set